Cardiovascular Qbank

Ace your homework & exams now with Quizwiz!

A 75-year-old man with angina pectoris has recurrent episodes of atrial tachycardia (240/min). A rapid sequence of normal QRS waves is seen on EKG. The episodes are controllable by the patient's performance of vagal maneuvers. Which of the following is the most likely etiology of this arrhythmia?

AV nodal reentry. This patient has paroxysmal supraventricular tachycardia (PSVT), which is a regular, rapid (150-250/min) arrhythmia originating in the atria or AV node. AV nodal reentry is the most common cause of this arrhythmia (about 70% of patients). In this condition, the AV node is pathologically divided into two functional pathways. The electrical impulse usually proceeds anterograde down the slow pathway and retrograde up the fast pathway. The P waves are recorded nearly simultaneously with the QRS complexes (which occur in rapid sequence) and are therefore obscured on EKG. This arrhythmia is commonly seen in older patients, about half of whom have underlying heart disease. Reentry PSVTs can be reverted to normal sinus rhythm by interrupting the reentry pathway. For example, the performance of vagal maneuvers often improves the condition by increasing AV nodal refractoriness

A 71-year-old man comes to the hospital with an episode of bright red blood per rectum. The patient reports that, a few hours ago, he passed a grossly bloody bowel movement. The passage was associated with some cramping lower abdominal pain. The patient's past medical history is significant for coronary artery disease and a myocardial infarction 3 years ago. He is poorly compliant with his beta-blocker and diuretic therapy, and his blood pressures have run around 140/85 mm Hg. While the patient is in the hospital, he has another episode of large volume, bright red blood per rectum. His blood pressure is 100/60 mm Hg while supine, and his pulse is 120/min. His hematocrit is 28%. He then begins to complain of substernal chest tightness radiating to his left shoulder. An electrocardiogram shows new T-wave inversions in the anterior leads. Which of the following is the most appropriate intervention at this juncture?

According to the available data, the patient is presenting with myocardial ischemia (as manifested by his angina) in the presence of anemia, which is caused by his acute bleeding secondary to diverticulosis. The appropriate treatment, therefore, is a blood transfusion, because he is not tolerating the acute blood loss.

Which of the following factors is recognized as contributing to atherosclerotic risk?

Decreased fibrinolytic activity or an elevated plasminogen activator inhibitor helps to modulate the balance between fibrinogen factors. Fibrinolytic activity helps to clear areas of plaque, which contributes to atherosclerosis.

A 72-year-old woman with no prior medical history comes to the emergency department because of a 3-hour episode of crushing substernal chest pain. The pain radiates to her arm and neck. An electrocardiogram reveals ST-segment elevation in leads II, III, and aVF. The patient has no obvious contraindication to anticoagulation. Which of the following is the treatment that should be currently pursued?

Administration of thrombolytic therapy, heparin, and aspirin. The patient is having an acute ST-segment elevation myocardial infarction. The infarct occurs as a result of an atherosclerotic plaque with thrombus formation, leading to coronary artery obstruction. Lysis with a thrombolytic agent has been shown to decrease mortality from early post-myocardial infarction. Aspirin prevents both platelet aggregation and reocclusion of the reperfused vessels. When aspirin and lytic therapy are given in the setting of ST-segment elevation MI, heparin is added to the regimen to stop new clots from forming.

An elderly homeless man who regularly resides outside the emergency department is acting strangely. He is often seen drinking alcohol and asking for money, but today he seems disoriented, confused, and weak. It is bitterly cold and there is snow on the ground, so one of the health care providers is concerned that the combination of cold weather and alcohol would predispose the patient to hypothermia. Two emergency medical technicians help bring the man inside so he can be fully evaluated. His temperature is 32.0°C (89.6°F), blood pressure 80/40 mm Hg, pulse 53/min, and respirations 10/min. Electrocardiogram would most likely show which of the following findings?

An upward deflection following the R wave (classically in lead II) describes an Osborn wave (or J wave of Osborn), which is pathognomonic of hypothermia. The Osborn wave appears on the EKG following ST-segment elevation and is seen when the patient's body temperature is 32-33°C (89-91°F). Although these findings are pathognomonic for hypothermia, their absence does not rule out heart involvement in a patient who has hypothermia.

A 28-year-old man seeks help for progressive edema affecting his right lower extremity. It first started about 2 years ago but has become much worse in the last 6 months. Physical examination shows pitting edema from the groin to the toes, as well as very prominent varicose veins and an area of chronic cellulitis above the medial malleolus. These findings are confined to the right side and are not present at all on the left. He also is noted to have a palpable thrill and an audible bruit over the right groin. His resting pulse is 115/min. He gives a history of having suffered a gunshot wound to his right groin 3 years ago, for which he had no specific therapy. Which of the following is the most likely diagnosis?

Arteriovenous fistula. The bruit and the resting pulse rate are virtually pathognomonic for an arteriovenous fistula, particularly in view of the history of untreated penetrating trauma to the groin. Significant venous hypertension can develop in that setting, leading to the other presenting symptoms.

Which of the following drugs antagonizes both the vascular and cardiac actions of norepinephrine (NE)? Atenolol, Esmolol, Labetalol, Metaproterenol, Prazosin

Norepinephrine (NE) is an agonist at alpha-1, alpha-2, and beta-1 receptors. NE exerts its vascular actions via alpha (predominantly alpha-1) receptors and its cardiac actions via beta-1 receptors. Labetalol is a selective antagonist at alpha receptors and nonselective antagonist at beta receptors and therefore could prevent all actions of NE.

A cyanotic infant is discovered to have a ventricular septal defect, an overriding aorta, right ventricular hypertrophy, and complete pulmonic stenosis. Which of the following accompanying congenital anomalies improves survival in this patient?

The ductus arteriosus connects the aorta with the pulmonary artery. If it remains patent after birth, it allows oxygenated blood to flow from the aorta to the pulmonary artery. In this patient, who has tetralogy of Fallot with complete right ventricular outflow obstruction, this anastomosis is a crucial source of blood to the pulmonary vasculature. Patients who have a patent ductus arteriosus have an advantage in the setting of tetralogy of Fallot. If tetralogy of Fallot is diagnosed in the newborn period, the infant may benefit from the use of the prostaglandin E1 agent alprostadil (Prostin VR Pediatric), which can be used to maintain the patency of the ductus arteriosus. A patent ductus arteriosus provides an alternative source of pulmonary blood flow while the infant has definitive surgery performed.

The mortality rate per year is greatest with disease of which coronary anatomy?

The left main coronary artery is the primary arterial supply to the left ventricle. Blockage of this artery leads to anterior wall and lateral wall myocardial infarction. Mortality for patients who have left main coronary artery disease is more than 10 times greater than in patients who have one- or two-vessel disease involving the other coronaries. Left main coronary artery is the proximal artery for both the left anterior descending and left circumflex arteries. Blockage of the left main artery will affect blood supply to the left ventricle, which is considered to be the workhorse for the entire heart. Blockage in this area affects more muscle mass in the heart than any other artery. Because of the high mortality associated with blockage in this artery, this artery is notoriously known as the "widow maker."

Which of the following diuretics acts at the distal tubule of the nephron? Ethacrynic acid, Furosemide, Hydrochlorothiazide, Mannitol, Spironolactone

The thiazide diuretics (e.g., hydrochlorothiazide, chlorothiazide, benzthiazide) promote diuresis by inhibiting reabsorption of NaCl, primarily in the early distal tubule. Thiazide diuretics act to inhibit distal convoluted tubule sodium and chloride reabsorption, causing diuresis and a drop in blood volume and blood pressure. They are very effective antihypertensive agents and are indicated as either first-line treatment for hypertension or preferred second-line agents.

A 68-year-old man is brought to the emergency department following a high-speed automobile accident. He is alert and complains of chest pain and mild back pain. His blood pressure is 80/60 mm Hg. Chest radiograph shows a widened mediastinum, tracheal deviation, bronchial displacement, and loss of the aortic knob. Which of the following is the most likely diagnosis?

This patient has a traumatic aortic rupture, which is the most common cause of immediate death following a motor vehicle accident. Deceleration causes twisting of the aorta and may lead to rupture. Up to 90% of individuals die at the scene, however a high index of suspicion and early intervention may save the remaining few. The symptoms include chest pain, back pain, and hypotension. Angiography is the gold standard and immediate surgery is mandatory. Radiographic findings include widening of the mediastinum, alteration of the aortic knob, pleural cap, tracheal deviation, bronchial displacement, and esophageal deviation.

Which of the following is an advantage of tissue plasminogen activator (tPA) over streptokinase for fibrinolytic therapy?

Tissue plasminogen activator is produced by and secreted from endothelial cells. Because it is not a foreign protein like streptokinase (derived from hemolytic streptococci), tPA is not allergenic. Both drugs can be used in the setting of acute myocardial infarction. Both drugs can result in hemorrhage, especially during prolonged therapy for treatment of pulmonary embolism or venous thrombosis. tPA is as much as 10 times more expensive than streptokinase. Both drugs result in the activation of plasminogen.

Valvuloplasty is indicated for mitral valve stenosis in which of the following scenarios?

Valvotomy is indicated in the symptomatic patient who has a mitral orifice less than 1.0 cm2/m2 BSA. Significant obstruction begins when the orifice is less than 2 cm2. Severe mitral stenosis with a valve opening of 1 cm2 would require a left atrial pressure of 25 mm Hg to maintain normal cardiac output.

A 2-month-old baby boy is brought to the clinic by his mother for a routine checkup. On physical examination, the health care provider hears a continuous machinery murmur near the left sternal border at the second interspace. What is the most likely diagnosis?

A continuous machinery murmur is characteristic of patent ductus arteriosus (PDA). PDA is also characterized by bounding pulses with wide pulse pressure. The ductus ateriosus is a structure that is necessary during gestation but becomes unnecessary after birth when the newborn takes a deep breath and the lungs begin to function. It normally closes within a few hours to days after birth. It most commonly fails to close in premature infants or in infants who are born to mothers living in high altitudes. If the ductus allows a large volume of blood to flow through, there is a wide pulse pressure that is identified along with bounding pulses. A continuous, machinery-sounding murmur is classically associated with this disorder. The murmur is continuous because the aortic pressure is higher than the pulmonary pressure during both systole and diastole. EKG may be normal or may show signs of ventricular hypertrophy. Indomethacin can be used to close the ductus and surgical ligation can be used if indomethacin fails. Diuretics may be used to treat extra volume states until more definitive closure can be performed.

A 15-year-old boy is concerned about multiple stretch marks that have developed on his chest, shoulders, and thighs over the previous 3 to 4 years. He plays center on his high school basketball team and is annoyed by the appearance of the skin lesions. His medical history is remarkable for myopia, upward lens displacement, and mitral valve prolapse. He is quite tall for his age and has long, thin extremities and fingers. He is accompanied by his father, who has the same tall stature and thin extremities. The skin lesions this young man is worried about are multiple striae in the pectoral, deltoid, and thigh regions. His chest is flattened on either side with forward projection of the sternum, and he has long, tapering fingers and toes. Which of the following is the most likely cause of his condition?

A genetic defect in the fibrillin gene. This patient has Marfan syndrome, an autosomal dominant inherited genetic defect of fibrillin. Abnormalities appear in 3 organ systems: the eye, the skeletal system, and the cardiovascular system. Skin manifestations may consist of striae (stretch marks). The skeletal features include long, narrow extremities and tapering fingers and toes. The patients are typically unusually tall. They may also have pectus excavatum, pectus carinatum, flat feet, and hyperextensibility of the knees and other joints. The ocular findings include long orbits causing myopia, and ectopia lentis (with upward displacement in 70%) sometimes causing acute glaucoma because of drainage obstruction secondary to lens displacement. Cardiovascular defects consist of weakness of the aortal intima leading to aneurysms and mitral valve prolapse. Acute dissection of the aorta is a common cause of death in these patients.

A 40-year-old woman is brought to the emergency department following a suicide attempt with imipramine (Tofranil). Her physical examination is otherwise unremarkable. On an electrocardiogram, which of the following abnormalities would most likely reflect possible cardiac toxicity?

A prolongation of the QT interval is highly predictive of both cardiac and CNS toxicities from tricyclic antidepressant ingestion. This medication has high lethality associated with its overdose because of its cardiac effects.

A 68-year-old man is brought to the emergency department with excruciating back pain that began suddenly 45 minutes ago. The pain is constant and is not exacerbated by sneezing or coughing. He is diaphoretic and has a systolic blood pressure of 90 mm Hg. There is an 8-cm pulsatile mass deep in his epigastrium, above the umbilicus. A chest radiograph is unremarkable. Two years ago, he was diagnosed with prostatic cancer and was treated with orchiectomy and radiation. At that time his blood pressure was high, for which he declined treatment. Which of the following is the most likely diagnosis?

Abdominal aortic aneurysms have a high incidence of rupture once they reach or exceed a size of 5.5 cm in males and 5.0 cm in females. Often the first manifestation is excruciating back pain, as the blood leaks into the retroperitoneal space before the aneurysm blows out into the peritoneal cavity. The combination of a big pulsatile mass and sudden severe back pain should always lead to this presumptive diagnosis. Looking for orthopedic or neurologic explanations can be a deadly mistake. In any patient who has a rupturing aneurysm or who is hemodynamically unstable, immediate surgical intervention is required.

A 63-year-old woman comes to the emergency department complaining of severe mid-abdominal pain. The patient reports that the pain has increased in intensity over the past few days. There has been no associated nausea or vomiting, no change in bowel habits, and no relief afforded by position changes. The woman is postmenopausal and does not take hormone replacement therapy. She has a 30-year history of hypertension and has been noncompliant with her calcium channel blocker and thiazide diuretic therapy. On examination, her abdomen is obese, but there is a suggestion of a nontender, pulsatile mass in the epigastric region. The remainder of the physical examination is normal. Imaging studies are done, and immediate surgical evaluation is indicated. Which of the following results were most likely obtained from the imaging studies?

Abdominal aortic aneurysms larger than 5 cm have greater than a 30% chance of rupturing in 3 years. As most ruptures result in death, and as the mortality of surgical repair has fallen sharply, surgical repair of the aneurysm is indicated. Aneurysms that are not repaired expand on average at about 0.4 cm per year. Repair of these aneurysms can be accomplished via either traditional open approaches or new endovascular approaches.

A 73-year-old man comes to the emergency department complaining of abdominal pain. He describes a dull, aching, constant pain in his mid-umbilical region. The pain has persisted over the past few days with increasing intensity, and it is not relieved by changes in position or eating. The patient has a past medical history significant for hypertension and coronary artery disease. He had a myocardial infarction 3 years ago. The man has moderate peripheral vascular disease with a prior femoral-popliteal bypass graft on the left. On physical examination, his blood pressure is 180/100 mm Hg and his pulse is 86/min. He has a loud S4 on chest examination. Abdominal examination shows a pulsatile mass in his abdomen and venous stasis changes bilaterally on his lower extremities. Which of the following is the most appropriate diagnostic test at this time?

Abdominal ultrasound (U/S) is the most cost-effective screening test for a suspected abdominal aortic aneurysm (AAA). Ultrasound of the abdomen is considered to be the definitive test to perform when AAA is suspected, as it has a sensitivity and specificity of almost 100%.

A 64-year-old woman comes to her primary health care provider for management of her hypertension, which has been treated unsuccessfully for several years. She was recently hospitalized for pulmonary edema, and an echocardiogram at that time showed a moderately depressed ejection fraction. She was diagnosed with congestive heart failure. Her medications include a thiazide diuretic and a calcium channel blocker. She has an allergy to furosemide. Her review of systems is positive for two-pillow orthopnea and occasional paroxysmal nocturnal dyspnea. On physical examination, her blood pressure is 150/80 mm Hg and her pulse is 80/min and regular. Her lungs are clear, and there are no extra heart sounds. Her extremities are without edema. Which of the following is the most appropriate management at this time?

Add ACE-I. This patient has both hypertension and congestive heart failure (CHF). An important concept to recognize in the treatment of medical conditions is that certain medications overlap syndromes and are efficacious in many areas. This "co-treatment" option maximizes each drug in a regimen and often addresses two or more issues simultaneously. In this case, ACE inhibitors have been shown to be very beneficial in prolonging the survival of CHF patients.

A 37-year-old African American man with a history of mild hypertension comes in for an annual examination. He has no complaints. He reports compliance with his low-salt diet. His only medication is hydrochlorothiazide, 25 mg each day. A thorough review of systems is negative. His temperature is 37.0°C (98.6°F), blood pressure 160/90 mm Hg, pulse 83/min, and respirations 10/min. Physical examination is within normal limits.

Add a second antihypertensive medication. This patient most likely has essential hypertension. Secondary forms of hypertension should only be suspected in those patients who develop hypertension at extremes of age (less than 25 or more than 55 years of age), who have suspicious physical examination findings, or who are resistant to multiple blood pressure medications. This patient is a healthy man who is only on one medication and does not have any laboratory or physical examination stigmata of secondary hypertension. Simply adding another medication to his current regimen is appropriate at this time.

A man is an unrestrained front-seat passenger in a car when it crashes; he sustains closed comminuted fractures of both femoral shafts. Shortly after admission, he develops a blood pressure of 80/50 mm Hg, pulse 110/min, and venous pressure of 0. He becomes pale, cold, and clammy. The rest of the physical examination and x-rays of the chest and pelvis are unremarkable. Sonogram of the abdomen performed in the emergency department is likewise negative. Which of the following most likely explains the patient's low blood pressure?

After extensive trauma to certain areas of the body, enough blood may accumulate to send the patient into hypovolemic shock. One of these areas is the femur (as seen in the present patient); the others are the thorax, abdomen, and pelvis. These patients should be treated with massive resuscitation and transfusion to overcome their hypovolemia until reduction and fixation can be performed. After extensive trauma, the thorax, abdomen, pelvis, or femurs have the potential to accumulate enough blood to produce hypovolemic shock. Intracranial bleeding does not lead to hypovolemic shock; there is not enough space inside the head for sizable blood loss to accumulate.

A 65-year-old patient has experienced several transient ischemic attacks over the past few months. Because his general health is poor, he is not considered an appropriate candidate for carotid endarterectomy. Examination reveals a regular rate on heart examination with a blood pressure of 130/86 mm Hg. The decision is made to treat him medically. Which of the following agents would be most appropriate for his therapy?

Although treatment of transient ischemic attacks (actually prophylaxis against stroke) remains a controversial area, you should be aware of current recommendations. Daily aspirin therapy has been shown in prospective, randomized studies to reduce the incidence of stroke and death in patients who have transient ischemic attacks. If alternative therapy is needed, either because the patient cannot tolerate aspirin or because aspirin therapy has failed, the antiplatelet agent clopidogrel (Plavix) can be used. Anticoagulation with coumadin does not decrease the risk for stroke and death any better than aspirin for patients who do not have atrial fibrillation, but it does increase the risk for intracerebral hemorrhage. When a patient develops a TIA, he or she is at extremely high risk for acute stroke situation. These patients need to have evaluation as to why they developed the TIA to address the underlying cause so that stroke can be prevented. If significant carotid stenosis is found, carotid endarterectomy is recommended. For patients who are not surgical candidates, stenting or angioplasty is recommended. Antiplatelet agents are preferred first-line agents in patients who have TIAs who do not have underlying atrial fibrillation or who do not have significant stenosis (<60%) post-TIA. For patients who have atrial fibrillation, warfarin or dabigatran (Pradaxa) are preferred nonsurgical therapies.

A 15-year-old girl is brought to a pediatric cardiology clinic with a complaint of chest pain. She states the pain has come and gone over the past year but has increased in frequency over the past few weeks. She describes it as a sharp pain over her left chest. Physical examination reveals a healthy appearing 15-year-old girl. Her temperature is 37.2°C (99°F), pulse 90/min, and respirations 20/min. Lung examination is normal. Cardiac examination reveals a late systolic murmur preceded by a click at the apex. No heave or rub is present. Electrocardiogram and chest radiograph are unremarkable. Which of the following is the most likely diagnosis?

An apical click followed by a late systolic murmur is classic for mitral valve prolapse. It can be a source of subjective chest pain in children. Mitral valve prolapse is more common in females. Antibiotic prophylaxis is no longer routinely recommended by the AHA prior to dental procedures unless prolapse is accompanied by severe regurgitation.

A young man sustains a gunshot wound to the base of his neck. He was shot point blank with a .38-caliber revolver. The entrance wound is above the left clavicle, below the level of the cricoid cartilage, and just lateral to the sternomastoid muscle. The exit wound is just above the spinous process of the right scapula. He has normal breath sounds on both sides, is awake and alert, is talking with a normal tone of voice, is neurologically intact, and is hemodynamically stable. Portable radiographs of the neck and chest taken in the emergency department show some air in the tissues of the lower neck but are otherwise non-diagnostic. Which of the following is the most appropriate next step in management?

Angiogram, esophagogram, esophagoscopy, and bronchoscopy prior to surgical exploration. Gunshot wounds to the base of the neck need exploratory surgery, but the exact approach and incision are determined by a more accurate knowledge of the location and extent of the injuries. If time permits, diagnostic studies thus should precede surgical intervention. The major vessels, the tracheobronchial tree, and the esophagus are the potential targets that have to be investigated. The esophagram needs to be performed with a water-based solution in case the patient has damage to the mediastinum, which would result in leakage from the esophagus and potential mediastinitis.

A 55-year-old man with hypertension and a past medical history of myocardial infarction is prescribed atenolol (Tenormin). This medication will lower his blood pressure by

Atenolol is a beta-adrenergic receptor blocking agent used in the treatment of hypertension. Medications in this drug class lower blood pressure by reducing both cardiac output and decreasing renin release from the kidney (to a lesser extent). Beta-blockers work as antihypertensive agents by blocking the sympathetic effect of the beta-adrenergic system. Blocking the beta-adrenergic system causes a lowering of the heart rate. The force of contraction of the ventricles is impeded and this results in a lower cardiac output. Because the heart is not contracting as strongly, there is less myocardial oxygen demand in the heart, which is one of the reasons that these agents are cardioprotective in patients who have heart disease. Because the heart is not contracting as strongly, cardiac output is lessened. Beta-blockers also cause an increase in smooth muscle contraction, which is why some patients who have reactive airways disease do not tolerate these agents because of an increased tendency for bronchospasm. Beta-blockers should not be used by patients who have peripheral vascular disease, as the impaired cardiac output will further decrease the delivery of blood into the peripheral circulation.

A 28-year-old man presents to his primary care provider for his yearly physical examination. He currently smokes approximately one pack of cigarettes a day and is interested in quitting. He has no significant past medical history and has no allergies. He is interested in smoking cessation classes but would like additional help. Which of the following medications would be the most appropriate agent for treating nicotine dependence in this patient?

Bupropion was approved by the U.S. Food and Drug Administration in 1996 as a treatment for nicotine dependence. The specific mechanism of action is unclear but is thought to relate to reducing craving for nicotine as well as reducing withdrawal symptoms after cessation. When used as a smoking cessation therapy, the medication is known as Zyban. This agent is an antidepressant that inhibits neuronal uptake of norepinephrine and dopamine.

A 23-year-old man sustains multiple stab wounds to his left chest. At the time of emergency department admission, his blood pressure is 70/45 mm Hg, pulse 110/min, and central venous pressure 4 mm Hg. He is diaphoretic and anxious, and he is notably pale. It is quickly determined that he has a left hemopneumothorax and so a chest tube is inserted. A total of 380 mL of blood is recovered initially, and another 120 mL is suctioned during the next hour. He is given 2 liters of Ringer's lactate, followed by 2 units of blood. Reassessment at the end of the first hour shows that his lung is expanded and his central venous pressure has gone up to 22 mm Hg, but his blood pressure is only 85/70 mm Hg, his pulse has gone up to 115/min, and he remains diaphoretic. Which of the following is the most likely diagnosis at this time?

Cardiac/pericardial tamponade is caused by the accumulation of blood within the pericardial space, creating an increase in the intrapericardial pressure. This in turn results in restricted cardiac filling with a resultant decrease in cardiac output. Shock with high central venous pressure in a chest trauma victim suggests either pericardial tamponade or tension pneumothorax. The latter has been ruled out by the re-expanded lung. The problem was not initially obvious because of concomitant bleeding but became clear once blood volume was restored. Patients who have this condition will have hypotension, jugular venous distention, and distant heart sounds (known as Beck triad).

Which of the following best describes why carotid sinus massage will result in the slowing of PSVT along with return of the patient to normal sinus rhythm?

Carotid massage of the baroreceptors will cause vagus nerve stimulation of the heart to increase, resulting in an increased parasympathetic action on the heart. This stimulation will slow the heart's conduction through the AV node. When the speed of the impulse is slowed through the antegrade pathway, it will break the reentry cycle of PSVT by slowing the antegrade pathway to a speed that approaches the speed of the retrograde pathway, thus terminating the essential requirement that the reentry dysrhythmia needs to work.

A 59-year-old woman comes to the health care provider because of recurrent episodes of lightheadedness upon getting up in the morning and occasionally upon standing up from a chair. She reports that on two occasions she has "passed out" soon after getting up from bed. Her temperature is 37°C (98.6°F), blood pressure is 130/80 mm Hg, pulse is 70/min and regular, and respirations are 14/min. She takes a beta-blocker and a thiazide diuretic for moderate hypertension diagnosed 6 months ago. She had rheumatic fever as a child. A thorough physical examination, including chest auscultation, is unremarkable. Which of the following is the most likely cause of this patient's symptoms?

Clinical history is crucial in the diagnostic approach to syncope. Furthermore, measurement of orthostatic blood pressures and pulses should be performed first with the patient in a supine position, and then checked again sitting and standing. This patient's episodes of lightheadedness and syncope can be best explained as an effect of antihypertensive therapy. Orthostatic hypotension is one of the most frequent side effects of antihypertensive drugs and should always be considered in the differential diagnosis of syncope of unexplained origin.

A patient admitted to the emergency department with chest pain is diagnosed with myocardial infarction. On discharge, the patient is prescribed aspirin but develops an allergic hypersensitivity reaction. Clopidogel (Plavix) is then prescribed. Which of the following is the mechanism of action of this drug?

Clopidogrel (Plavix) is an antiplatelet drug that inhibits adenosine diphosphate-induced platelet aggregation and interacts with platelet glycoprotein IIb/IIIa, a fibrinogen receptor that links platelets together and leads to platelet plug formation. The drug thus prevents fibrinogen from binding to platelets. The drug is useful both for patients who have coronary disease and for those who have cerebrovascular disease such as stroke. Of course, hemorrhagic stroke should first be ruled out before it is deemed safe to use any anticoagulant. Clopidogrel is used in both acute coronary syndrome (for patients not having an open bypass) and for thrombotic event prevention. For stroke prevention, it is used when aspirin or warfarin (Coumadin) cannot be used. Antiplatelet agents have been shown to be effective in the setting of acute coronary syndrome and for prevention of thromboembolic events. Aspirin is commonly used in these settings, but for patients who have a contraindication for aspirin use, clopidogrel (Plavix) is a useful alternative. The primary side effect with these antiplatelet agents is bleeding. There is a Black Box warning for clopidogrel for patients who are poor metabolizers of this medication; if they cannot metabolize clopidogrel appropriately, they wil have diminished efficacy with its use. Genotype testing is available to identify these patients.

A 22-year-old construction worker is brought to the emergency department after being pinned by a steel beam that fell on his legs and mid-torso. The patient was brought to the hospital by the EMS team who had extricated him. At the scene, his Glasgow score was 15/15, and he was alert and oriented. On arrival the patient appears pale and bloody and in a significant amount of pain, but still alert. Examination reveals bilaterally crushed lower extremities with a visibly pulsating bleed.

Crush injuries such as the one sustained by this patient often result in a massive release of muscle contents. Creatine kinase (CK) is an enzyme released by dead or damaged muscle into the blood. By itself it is harmless, but it is a marker for myoglobin, which is directly nephrotoxic. Much data exist showing early intervention with copious alkalinized IV crystalloid can prevent renal damage.

On physical examination of a patient, the health care provider hears a rumbling mid-diastolic murmur with an opening snap heard best at the apex of the heart. What is the most likely explanation for this finding?

mitral stenosis

A 71-year-old retired physician complains of increasing calf pain when walking uphill. The symptoms have gradually increased over the past 2 months. His past medical history is significant for a transient ischemic attack 6 months ago. Over the past month, his blood pressure has worsened despite previous control with diltiazem, hydrochlorothiazide, and propranolol. His other medications are isosorbide dinitrate and aspirin. On physical examination, his blood pressure is 152/90 mm Hg and his pulse is 66/min. There is a right carotid bruit. His lower extremities are mildly cool and have diminished pulses at the dorsalis pedis. An ACE inhibitor is added to help control his blood pressure. Which of the following is the most appropriate in the management of his leg pain?

Decrease propranolol (Inderal) dose. Propranolol, a beta-blocker, is considered to be relatively contraindicated in patients who have peripheral claudication. Although beta-blockers are excellent medications in patients who have hypertension, the beta blockade may predispose to arterial vasoconstriction peripherally. In addition, beta-blockers are negative inotropic agents that decrease the force of contraction of the heart. Because the heart is not contracting as strongly, there is less blood being propelled into the peripheral vascular system. The dose of beta-blockers in this patient should therefore be reduced or gradually eliminated.

A 59-year-old man with a history of myocardial infarction presents to his health care provider complaining of shortness of breath. On examination, his pulse is 110/min and his respiratory rate is 22/min. He has rales in both lung fields, a normal sinus rhythm with an S3 gallop, and 2+ pitting ankle edema. A chest radiograph reveals cardiomegaly, and his ejection fraction on echocardiogram is calculated at 37%. Which of the following medications would alleviate this patient's symptoms by significantly reducing both the preload and afterload on the heart without affecting its inotropic state? Digoxin, DIltiazem, Enalapril, Furosemide, Propranolol

Everything in this case points to congestive heart failure (CHF): dyspnea, elevated heart rate, S3, peripheral edema, and reduced ejection fraction. A drug that will alleviate the symptoms by decreasing both preload and afterload is necessary. Enalapril is an angiotensin-converting enzyme (ACE) inhibitor. ACE inhibitors (captopril, enalapril, lisinopril, benazepril, ramipril) work by blocking the conversion of angiotensin I to angiotensin II in the lungs. Angiotensin II is both a potent vasoconstrictor and a stimulator of aldosterone production. Aldosterone acts by promoting sodium (and thus water) reabsorption by the kidney. An ACE inhibitor will therefore promote vasodilatation (reducing afterload), as well as reduce intravascular volume (decreasing aldosterone, as less renin is being released). ACE inhibitors are first-line agents for patients who have heart failure because they act as vasodilators. ACE inhibitors decrease afterload, which allows the heart to have improvement in its ejection fraction. They do not have an inotropic or chronotropic effect on the heart. There is a mortality benefit demonstrated with the use of these agents. Beta-blockers and the aldosterone antagonist, spironolactone, also have been shown to have an improvement in mortality in patients who have chronic heart failure.

A 29-year-old man is brought to the emergency department in a comatose state a few hours after complaining of sudden onset of excruciating headache. His friend does not know if the patient has any underlying medical conditions. Neurologic examination reveals dilated pupils poorly responsive to light. A CT scan of the head without contrast demonstrates hyperdensity within the suprasellar cistern, whereas MRI scan is unremarkable. Lumbar puncture shows hemorrhagic cerebrospinal fluid. Which of the following is the most likely diagnosis?

Headache of sudden onset ("thunderclap" headache), rapid deterioration of mental status, and blood in the CSF are virtually diagnostic of ruptured berry aneurysms. Note the characteristic hyperdensity on CT scan of the suprasellar cistern, indicating blood in the subarachnoid space. Rupture of a berry aneurysm is the most common cause of subarachnoid bleeding. Berry aneurysms develop as a result of congenital weakness at branching points of the arteries in the circle of Willis. These outpouchings tend to expand progressively, but in most cases they remain asymptomatic. Hypertension facilitates development and rupture of berry aneurysm. One third of patients recover, one third die, and one third develop re-bleeding. Rapid onset of coma is an ominous sign.

After an accident at work resulting in severe hemorrhage, a machinist is rushed to the emergency department. Which of the following sets of autonomic responses would be predicted in this patient? (BP & HR)

Hypotension & Tachycardia.After a decrease in blood pressure (e.g., after a hemorrhage), one would expect an increase in sympathetic outflow and decrease in parasympathetic outflow. As a result of the hemorrhage, there would be less blood in the body to circulate, with resultant increase in heart rate, decrease in both blood pressure and gastrointestinal motility, and dilation of the pupils.

A 59-year-old man who is scheduled for an abdominal aortic aneurysm (AAA) repair in 3 weeks presents to the health care provider's office. The patient's AAA was diagnosed last week via ultrasound. Imaging at that time revealed a 5.5-cm aneurysm of his abdominal aorta extending bilaterally into his iliac arteries. The patient also has moderate hypertension, with a mean daily blood pressure of 150/95 mm Hg. On physical examination, the patient appears in no distress. He weighs 274 pounds and is 5 feet 9 inches tall (BMI is 40.5). His lungs are clear, and he has a loud S4. His AAA is palpable as a pulsatile mass in his abdomen. He complains of very mild back pain. Which of the following is the most appropriate intervention to prepare this patient for surgery?

Improve blood pressure control. The most important interventions involve lessening the chance for rupture of an abdominal aortic aneurysm (AAA). Other interventions include counseling for smoking cessation and improving blood pressure control. Abdominal aortic aneurysm rupture typically has devastating consequences, which is why elective repair is indicated when the aneurysm size is 5.5 cm in males or 5.0 cm in females. Once an AAA is identified, patients are assessed for factors that are known to increase the risk for rupture, such as uncontrolled hypertension, smoking, and trauma to the abdomen. Beta-blockers are typically given, as they are known to be cardioprotective and because they will lower blood pressure. Beta-blockers have been shown to reduce morbidity and mortality for high-risk patients undergoing major noncardiac surgery, so these should be given to patients who have AAA unless there is a contraindication for their use.

A 52-year-old man is discharged from the hospital after an uncomplicated myocardial infarction. Several weeks later, he visits his primary care provider complaining of insomnia, anorexia, and depressed mood. He appears to be clinically depressed. He denies any current chest pain or shortness of breath. Which of the following would be the most appropriate medication to initiate for this patient?

In a patient who has cardiac complications, a selective serotonin reuptake inhibitor (SSRI) such as sertraline has been demonstrated to be the safest and most effective medication for the treatment of clinical depression. It has overtaken tricyclics in the management of depression in patients who have pre-existing heart disease, because it is not associated with prolongation of the QT interval that the tricyclics are known to cause. When compared to tricyclics, SSRIs also have a safer profile if a patient overdoses.

A 43-year-old woman comes to the health care provider with a several-month history of dyspnea on exertion. She denies chest pain or a family history of coronary artery disease. She has a history of rheumatoid arthritis. Chest radiograph is remarkable for calcification of the heart border on the lateral film. She appears to have constrictive pericarditis. Which of the following physical examination findings would most likely be expected in this patient?

In constrictive pericarditis, the lateral view of the chest may demonstrate calcification of the anterior pericardium. This may be seen in 50% of patients who have longstanding constriction. The pericardial thickening may be seen on an echocardiogram. A pericardial knock is heard after the aortic valve closes, which is caused by the sudden cessation of ventricular filling.

A 60-year-old woman with a long history of hypertension comes to the emergency department with a complaint of increasing and recurrent shortness of breath with minimal exertion. She is noted to have a prominent precordial impulse, and a chest radiograph reveals a prominent left ventricular shadow. A stress test is negative for ischemia. She is found to have left ventricular hypertrophy and a normal ventricular ejection fraction on echocardiogram. An S3 is heard on auscultation of the heart. Which of the following is the most likely underlying diagnosis causing this presentation?

Increased resistance to filling of one or more cardiac ventricles has been termed diastolic heart failure and can produce increased pulmonary capillary wedge pressures and respiratory complaints. In myocardial hypertrophy, impaired diastolic relaxation occurs. The key to differentiating systolic from diastolic types of heart failure is in the assessment of ejection fraction; it is normal and preserved with diastolic heart failure and impaired with systolic.

A 67-year-old man comes to the clinic complaining of steady, dull back pain over the past 3 weeks. He states that he has recently moved after retiring from a career in banking and is searching for a new health care provider. His past medical history is significant for diverticulosis, prior smoking, and hypertension. He says that he has run out of his blood pressure medication. He denies trauma to his back and otherwise feels well. On physical examination, his blood pressure is 170/93 mm Hg with a pulse of 88/min. He has no tenderness over the spinal processes or paraspinal areas. His abdomen is obese, but there is a suggestion of a non-tender, pulsatile mass in the epigastric region. The remainder of the physical examination is normal. Which of the following diagnoses should be considered at this time?

It is imperative to recognize the potential presence of an abdominal aortic aneurysm (AAA). The combination of the history of hypertension and smoking, the new back pain, and a pulsatile mass on examination is highly suggestive for abdominal aneurysm. Additionally, he is a male older than 65 years of age, which also puts him at higher risk for AAA development. The back pain occurs as the expanding mass compresses structures in the retroperitoneum. It is particularly important to make the diagnosis, because large aneurysms (greater than 5 cm in diameter) are associated with a very high risk for rupture and subsequent mortality.

A 58-year-old man with history of atrial fibrillation is prescribed warfarin to prevent clot and embolism formation. His prothrombin time (PT) is regularly monitored. Administration of which of the following drugs would result in an increase in his PT and require readjustment of his warfarin dosage? Rifampin, Carbemazepine, Ketoconazole, Phenytoin

Ketoconazole. This question is testing a favorite exam concept—the relationship between the hepatic cytochrome P-450 system and the metabolism of drugs. Certain drugs can affect the metabolism of other drugs by either inducing or inhibiting hepatic microsomal enzyme activity. This means that the amount of circulating warfarin increased and that you are looking for a drug that inhibits the P-450 system (thereby decreasing warfarin metabolism). Of the drugs listed, only ketoconazole inhibits the P-450 system. Other important inhibitors include cimetidine (Tagamet) and isoniazid (INH).

A 63-year-old white man who has recently retired from work as a plumber for over 30 years returns to his health care provider saying that he has been feeling very down lately, and has been having decreased appetite and a loss of interest in activities that used to give him pleasure. He is a smoker, drinks no alcohol, and is being treated by his health care provider for moderate essential hypertension. Physical examination is unchanged from his previous visits. Which of the following is the most appropriate next step in management?

Many medications used to control hypertension, such as propranolol, and in the past, reserpine, are known to occasionally lead to depressive symptoms. By evaluation of the patient's medication record, the physician can evaluate which medications were started at what time and can make adjustments to dosage or switch to alternative medications to control hypertension. Changing the antihypertensive medication will possibly improve the depressive symptoms without the need to start an antidepressant.

Patients having dilated cardiomyopathy will have which of the following changes on the EKG?

Low amplitude of the QRS complexes. In dilated cardiomyopathy the EKG often shows sinus tachycardia, or atrial fibrillation, ventricular arrhythmias, left atrial abnormality, diffuse nonspecific ST-wave abnormalities, intravascular conduction defects, and low voltage. The low voltage is caused by the impaired cardiac function of the heart because of the cardiomyopathy. Because the EKG measures the electrical strength of contraction, dilated cardiomyopathy will show impairment of the strength of contraction as the cardiac myocytes become less functional.

A 55-year-old man with no known cardiac history comes to the emergency department complaining of crushing substernal chest pressure that began 20 minutes prior to his arrival. He took sildenafil earlier in the evening prior to sexual intercourse. His past medical history is significant for bilateral inguinal hernia repairs 6 weeks ago, remote peptic ulcer disease, and prior cocaine and heroin addiction. On examination, he is diaphoretic and appears anxious. His blood pressure is 150/75 mm Hg with a pulse of 100/min. An electrocardiogram (ECG) obtained while he had severe chest pain revealed tall positive T waves and 1-mm elevation of the ST segments in leads V2 through V5. A serum creatine kinase (CK) drawn on presentation returned at 85 U/L (normal <250 U/L). Which of the following is the most likely diagnosis?

MI. The combination of substernal chest pressure with ECG findings of hyperacute T waves and ST elevation in this clinical setting suggests early anterior myocardial infarction. The myocardial damage causes time-dependent effects on the electrical properties of the myocardial cells. The earliest electrical evidence of myocardial injury is often development of hyperacute T waves, followed by elevation of the ST segments, inversion of the T wave, return of the ST segments to normal, and finally, the development of Q waves. It does not matter that the CK, a marker of myocardial injury, is not elevated, as it takes several hours for the CK to appreciably rise after myocardial damage has occurred. Myocardial infarction is diagnosed by having positive findings in at least two out of the following three criteria: typical symptoms of chest pain, EKG changes (>1 mm ST-segment elevation or depression in two or more contiguous leads, T-wave inversions), and elevations in cardiac markers. There is no need to wait for the cardiac markers to be elevated to diagnose myocardial infarction. The markers take at least 4 hours to become elevated. Treatment can be based on the classic EKG findings and patient history alone when deciding whether an ST-segment elevation MI is occurring. There are definite time limitations when considering thrombolytic therapy, although that window continues to increase in the setting of continued chest pain on the part of the patient.

During the performance of a supraclavicular node biopsy under local anesthesia, a hissing sound is suddenly heard, and the patient suddenly dies. At the time of the catastrophic event, the target node was under traction, and the final cut was being made blindly behind it to free it up completely. The patient, an otherwise healthy 24-year-old man, was inhaling at that moment. Which of the following most likely caused this patient's death?

Major vein injury with air embolism . Major veins at the base of the neck have negative pressure during inspiration and, if injured at that moment, will suck air rather than bleed. The air embolism then leads to sudden death.

Which of the following medications is the most appropriate choice for the treatment of hypertension during pregnancy? Lisinopril, Atenolol, Verapamil, Furosemide, Methyldopa

Methyldopa, an alpha-2 adrenergic agonist, decreases blood pressure by decreasing peripheral vascular resistance. It is the drug of choice for treating hypertension during pregnancy that may lead to pre-eclampsia, which is the most common cause of maternal and fetal morbidity and mortality. It acts on the central circulation as an alpha-agonist. It is listed as category B for use in pregnancy, which maintains that it has been proven safe for use in the first trimester of pregnancy.

A 57-year-old man comes to his health care provider for a preoperative evaluation. He has been a long-time patient in this office and has been treated for hypertension and gastritis. He has been scheduled for an elective cholecystectomy in 2 days because of ongoing gallbladder symptoms. He currently takes omeprazole (Prilosec) for his gastritis and thiazide for his hypertension daily. He smokes two packs of cigarettes per day. His home blood pressure log shows that his systolic pressures range from 150 to 190 mm Hg, and his diastolic pressures range from 80 to 105 mm Hg, indicating that his blood pressure may be not adequately controlled for the surgical procedure. Which of the following medications is most appropriate in the perioperative period for added blood pressure control?

Metoprolol. There is an extensive body of literature indicating that beta-blockers given to non-cardiac surgical patients who are at risk for cardiac events are associated with a more favorable outcome in terms of postoperative cardiovascular morbidity and mortality. This patient has somewhat poorly controlled hypertension, as well as at least three cardiovascular risk factors (hypertension, tobacco, age). Ideally one would like to have better control of the blood pressure and to reduce any risk for adverse perioperative events before the patient has elective surgery. Beta-blockers can achieve both of these endpoints. These agents are available parenterally and can be given on a dose titration basis. Although the most recent blood pressure guidelines do not list beta-blockers as first-line agents for blood pressure control, these agents may have utility in the perioperative setting because of their cardiac morbidity benefit.

A 21-year-old professional dancer complains of several episodes of near loss of consciousness during a performance. She has been in excellent health and is a principal dancer in the New York City Ballet Corps. She has no family history of coronary artery disease. She does not smoke, and a recent cholesterol profile was normal. On physical examination, her blood pressure is 142/88 mm Hg and her pulse is 84/min and regular. She has a brisk carotid upstroke with a double impulse palpable. She has a loud S4 and a harsh systolic murmur heard along the left sternal border. The murmur is accentuated during the Valsalva maneuver and when she stands from a squatting position. An electrocardiogram reveals severe left ventricular hypertrophy. Which of the following is the most appropriate medication in the management of this patient?

Metoprolol. This patient has the presentation of hypertrophic cardiomyopathy, a frequent cause of syncope or near syncope in young patients. She has a systolic ejection murmur that is heard at the lower left edge of the sternum, which is exacerbated by exercise and standing and lessened by lying supine or squatting. This murmur can be distinguished from other systolic murmurs by its increase with the Valsalva maneuver or standing from a squatting position. Any maneuver that acts to decrease venous return of blood to the heart and left ventricular size will increase the murmur of hypertrophic cardiomyopathy, because the obstructive component increases as the left ventricular cavity shrinks. Beta-blockers such as metoprolol help to slow ventricular rate, thereby increasing ventricular filling time. When there is increased time for ventricular filling, there will be more blood in the left ventricle and the patient's obstruction will be lessened. This patient is at risk for sudden cardiac death and should be kept from exercising, which may decrease blood volume and which can have accentuated venous blood pooling when the patient rests from exercise. An implantable cardiac defibrillator may also be used to treat the patient's sudden cardiac death should it occur. All affected patients need to refrain from high intensity exercise.

A 7-year-old girl is brought to the office because of fever, headache, and malaise that has been increasing over the previous 7 days. She had been away at her grandparents' house in the countryside for vacation and had come down with a sore throat and fever. Her grandmother had given her some over-the-counter cold medication and kept her in bed for a few days, but when she had not been getting better, they had brought her back to the city. Her mother noted a faint red rash that has been waxing and waning on her chest, back, and shoulders, that would become very red after a hot bath. The girl's past medical history is unremarkable and she has not been on any medications recently. On physical examination, the patient is in moderate distress with a temperature of 39.4°C (102.9°F), pulse of 120/min, and respirations of 30/min. She complains of chest discomfort and repeatedly coughs during the examination. Deep breaths cause moderate chest pain, so her breathing is shallow. She also has difficulty swallowing. The tonsils are edematous, erythematous, and covered with purulent exudate. Her ankles are swollen, erythematous, and with limited range of motion. The rash her mother described is present only on the right shoulder in the form of an annular erythematous plaque, 5 cm in diameter. A rapid strep test done in the office is positive. Which of valvular disease is most likely to develop in this patient?

Mitral Valve. This patient has acute rheumatic fever with signs of evolving carditis. Mitral valve disease is the most common complication, occurring in up to 70% of cases. Acute rheumatic fever is caused by infection with group A β-hemolytic streptococci. It is most commonly seen in children aged 5-15 years because they are most susceptible to infection with group A β-hemolytic streptococci. Skin infections with group A β-hemolytic streptococci do not predispose to rheumatic fever, but upper respiratory infections do. Acute rheumatic fever usually presents 1-5 weeks after a preceding streptococcal pharyngitis. Diagnosis is based on the Jones criteria that include clinical signs and symptoms. Acute rheumatic fever is diagnosed if there are two major criteria, one major and two minor criteria plus preceding streptococcal infection, and chorea without any other underlying cause. If arthritis is one of the presenting major criteria, then arthralgia cannot be a presenting minor criterion. Polyarthritis is very common and may be the earliest manifestation of rheumatic fever. It commonly involves the large joints of the lower extremities (knees, ankles) and is migratory in character. Aspirin readily suppresses the signs and symptoms and prevents further migration. Erythema marginatum occurs in up to 13% of patients as 1- to 3-cm pink macules and papules and annular plaques that wax and wane within hours and are greatly accentuated by heat. Subcutaneous nodules are infrequently found on the extensor surface of the extremities. Management of acute rheumatic fever begins with treatment of the streptococcal infection and monthly penicillin prophylaxis. Salicylates help control the arthritis and also treat carditis without failure. Steroids are used when there is carditis with heart failure, and heart failure is treated in the conventional manner. Valvular disease is the most common complication of rheumatic fever. In order of frequency, the mitral, aortic, tricuspid, and pulmonary valves may be involved. Mitral insufficiency presents as a new murmur that is apical, pansystolic, high-pitched, and of a blowing character. It radiates to the left axilla and is grade 2/6 or greater. Pancarditis is the second most common and the most serious complication of acute rheumatic fever, presenting as dyspnea, mild to moderate chest discomfort, pleuritic chest pain, edema, cough, or orthopnea. Rheumatic fever can be prevented by administering penicillin within 9 days of becoming infected with group A β-hemolytic streptococci. Rheumatic fever is most common in developing countries, especially in the tropics.

A 55-year-old woman has been known for years to have mitral valve prolapse. She has now developed exertional dyspnea, orthopnea, and atrial fibrillation. She has an apical, high-pitched, holosystolic heart murmur that radiates to the axilla and back. Because of her deterioration, surgery has been recommended. Which of the following is the most appropriate procedure?

Mitral valve annuloplasty. The physical findings are classic for mitral valve insufficiency. Whenever possible, repair of the native mitral valve is preferable to replacement. The way to repair an insufficient valve is to tighten the annulus, bringing the leaflets closer to one another, allowing better coaptation of the valve. Repair of the mitral valve, rather than replacement, allows better preservation of the left ventricle and no need for long-term anticoagulation. Mitral insufficiency (regurgitation) is associated with a holosytolic blowing murmur at the apex of the heart that radiates into the axilla. Mild to moderate disease can be asymptomatic for years, but as it progresses there is eccentric cardiac hypertrophy with increased left ventricular end diastolic volume. This volume overload leads to left ventricular dysfunction. Mitral valve prolapse is a risk factor for the development of mitral regurgitation. Treatment of mitral regurgitation will depend on whether the condition is acute or chronic. For chronic cases, treatment will depend on the left ventricular function; with an adequate left ventricular ejection fraction, acceptable forms of treatment are ACE inhibitors, beta-blockers, diuretics, and surgery, which would include valvuloplasty or annuloplasty (preferentially) or mechanical valve replacement with or without anticoagulation (depending on whether it is a mechanical valve or bioprosthetic valve). Mitral valve repair is preferred over replacement because it is able to better support left ventricular function.

A 55-year-old man comes to the emergency department complaining of chest pain that radiates to his left arm and shortness of breath for the past hour. Which of the following cardiac enzymes is the most sensitive for acute muscle damage? CK-MB, LDH, Myoglobin, AST, Troponin-T

Myoglobin identifies any muscle damage but is so sensitive that it is not routinely used in the evaluation of myocardial infarction. It is heme protein that is mainly found in muscle. It rises in 2-3 hours and is rapidly cleared by the kidneys. It may be used as a "rule out" for patients presenting urgently to a health care facility because of its sensitivity; if myoglobin is normal, there is little chance that any significant muscle damage is present. Any muscle damage in the body will cause myoglobin levels to increase. Patients suspected of having myocardial infarction typically will have only the CK-MB and troponin levels drawn. Myoglobin test could be used but rarely is in the setting of suspected cardiac damage because it is too sensitive at identifying muscle damage. Troponin I is more specific than troponin T because it is not affected by chronic kidney diseaseMost providers will primarily rely on troponin I to help risk-stratify patients who present to an urgent care setting with complaints related to the chest.

A 63-year-old woman comes to the emergency department complaining of chest pain. The patient states that the pain began during her morning walk. It started as a dull pressure over her breastbone and then radiated to her left arm. Over the next few minutes, it escalated in intensity and was not relieved by rest. She called 911 and was brought to the emergency department. Her past medical history is significant for hypertension and hyperlipidemia. Her medications include atenolol and simvastatin daily. On physical examination, her blood pressure is 190/100 mm Hg, and her pulse is 60/min. Which of the following is the most appropriate agent to lower her blood pressure? IV Norepinephrine, IV Nitroglycerine, PO HCTZ, PO Metoprolol, PO Furosemide

Nitroglycerin is a potent vasodilator that acts predominantly on venous compliance to reduce preload to the heart. When administered intravenously, it can be rapidly titrated to produce optimized blood pressure control. IV nitroglycerin is effective when given in the setting of unstable angina. Oral forms of hydrochlorothiazide and metoprolol are excellent first-line antihypertensives with proven mortality benefit. The onset of their antihypertensive activity is relatively slow and therefore not appropriate for this case, in which blood pressure control must be achieved quickly. IV beta-blockers may be used to lower the blood pressure in an expeditious manner. The combined alpha/beta-blocker labetolol (Trandate) is a first-line agent for lowering blood pressure when given intravenously. Patients who have abrupt hypertension and angina symptoms need to have blood pressure lowered in an expeditious way. Nitroglycerin is the preferred agent in the setting of hypertension and angina because of its ability to drop the preload, vasodilate, and dilate the coronary arteries. Parenteral agents are preferred because of their rapid onset of activity and ability to be titrated. Various parenteral agents are approved in this setting, including sodium nitroprusside (Nipride) and labetolol (Trandate).

A patient who is being treated for hypertension related to a myocardial infarction that occurred 2 hours ago is medicated with IV nitroprusside (Nipride). Which of the following is the expected action of this drug?

Nitroprusside is a very useful IV agent that causes dilatation of both arterioles and venules. It has a very rapid onset of action and is typically used in an emergency department or intensive care unit situation. It can be titrated to effect on a drip-to-drip basis. It is used in settings in which the blood pressure needs to be carefully controlled. It is listed on ACLS protocols as one of the treatments (along with the combined alpha- and beta-blocker labetolol) for hypertensive emergency/hypertensive urgency. Nitroprusside can improve perfusion of vital organs and reduce the workload of the heart because it is both an arteriolar and venodilator. Problems sometimes encountered with this drug include hypotension (best avoided by starting with a low dose and continuously monitoring systemic arterial and pulmonary capillary wedge pressures) and accumulation of toxic metabolite (thiocyanate) of cyanide in patients who have liver or renal failure. Many physicians prefer to use IV nitrate rather than nitroprusside because of its lesser toxicity.

A 1-day-old infant girl appears dusky in the newborn nursery during feeding. Oxygen is immediately administered by nasal cannula. Shortly afterward, she develops tachypnea. On physical examination her blood pressure from the right upper arm is 50/30 mm Hg, pulse 180/min, and respirations 60/min. Echocardiogram is consistent with hypoplastic left heart syndrome. Which of the following would likely be found on auscultation?

No murmur, precordial hyperactivity, loud second heart sound. Hypoplastic left heart (HLH) syndrome is a group of closely related cardiac anomalies characterized by underdevelopment of the left cardiac chambers, atresia or stenosis of the aortic and/or the mitral orifices, and hypoplasia of the aorta. These anomalies are an especially common cause of heart failure in the first week of life. The left atrium and ventricle often exhibit endocardial fibroelastosis. Pulmonary venous blood traverses a patent foramen ovale, and a dilated and hypertrophied right ventricle acts as the systemic as well as pulmonary ventricle; the systemic circulation receives blood by way of a patent ductus arteriosus. Infants who have HLH syndrome develop poor perfusion, and metabolic acidosis when systemic blood flow decreases. When the ductus closes, inadequate blood flow to the body occurs because the ductus is the only path for blood to flow from the right ventricle to the body. Even if the ductus remains open, when the infant is given oxygen, the oxygen will dilate the vasculature of the pulmonary circulation, and blood will preferentially flow to the lower pressure pulmonary system, depriving the systemic circulation of adequate perfusion. ECG usually shows right axis deviation, right atrial and ventricular enlargement, and nonspecific ST- and T-wave abnormalities in the left precordial leads. Chest radiography may show only slight enlargement shortly after birth, but with clinical deterioration there is marked cardiomegaly with increased pulmonary vascular markings. Echocardiography is diagnostic and will show a diminutive aortic root and left ventricular cavity and absence or poor visualization of aortic and mitral valves. An infant who has HLH syndrome has a hyperdynamic precordium because the enlarged right ventricle is contracting against systemic pressure. The infant also has a loud second heart sound (S2) because the pulmonary artery acts as the aorta by pumping blood to the systemic circulation through the ductus arteriosus. The high end-systolic pressure markedly enhances S2. The flow from the right ventricle to the pulmonary artery is not turbulent; therefore, there is usually no significant murmur heard on auscultation. When the ductus closes, or when the pulmonary vasculature resistance falls, the flow to the systemic circulation will decrease, causing greatly diminished peripheral pulses. Management of the hypoplastic left heart syndrome patient includes infusion of prostaglandin E1 and administration of room air while on a ventilator (instead of oxygen, which will dilate the vasculature of the pulmonary circulation, causing blood to flow into the pulmonary system rather than the systemic circulation). Prostaglandin E1 may open the ductus arteriosus and restore systemic blood flow. Administration of room air or even hypobaric oxygen (FiO2 less than 21%) and the use of muscle relaxants can prevent hyperventilation and subsequent pulmonary vasodilation, thus reversing systemic hypoperfusion and metabolic acidosis.

A 14-year-old boy is brought to the health care provider with decreased exercise tolerance. He is up to date on all of his childhood immunizations and has been generally healthy until now. He is noted to have a grade III/VI systolic ejection murmur best heard at the left upper sternal border and a grade II/VI mid-diastolic murmur at the lower left sternal border. The first heart sound is normal. The second heart sound is widely split and fixed. A right ventricular impulse is palpated. On a chest roentgenogram, the pulmonary artery segment is enlarged, and pulmonary vascular markings are increased. An electrocardiogram shows right axis deviation. Which of the following congenital heart diseases does this patient most likely have?

One of the most common types of structural congenital heart disease to present in adolescence is atrial septal defect (ASD), and the most common presentation is a heart murmur. Some patients, however, present with arrhythmias, decreased exercise tolerance, or a paradoxic embolus. The physical examination can show classic findings of an ASD; in some cases, however, the findings may be extremely subtle. The murmur associated with the ASD is not caused by blood flow traversing the actual defect but rather by the increased volume of blood flow across the pulmonary valve and, to a lesser extent, across the tricuspid valve. The murmurs of an ASD thus are a systolic ejection murmur at the upper left sternal border and a mid-diastolic murmur at the lower left sternal border. The second heart sound is widely split and fixed with regard to respiration. The S2 split is caused by the delay of the pulmonic valve to close because of the increase in blood volume on the right side of the heart. On palpation, a right ventricular impulse is present. The chest radiograph shows evidence of an enlarged pulmonary artery segment in the posteroanterior projection. The superior vena cava shadow may not be visible because of the rotation of the heart secondary to right ventricular volume overload. Pulmonary vascularity is increased, and the heart may be somewhat enlarged. The lateral projection shows the right ventricular enlargement with filling of the retrosternal airspace. The ECG has a normal to rightward axis and a right ventricular volume overload pattern in the precordial leads.

A 16-year-old boy is brought to the urgent care clinic with a temperature of 38.4°C (101°F) and low back, wrist, and knee pain. He had a sore throat 1 month earlier. His arthritis is diffuse. Pea-sized swellings are noted over the skin on his knees. He has a serpiginous erythematous area on his anterior trunk. His blood and throat cultures are negative, and his CBC is unremarkable. His antistreptolysin-O (ASO) titer is high. Which of the following is the most appropriate therapy?

PCN and ASA. This patient has acute rheumatic fever from group A streptococci. He has migratory polyarthritis, erythema marginatum, and subcutaneous nodules. Other features absent in this patient are chorea and carditis. His ASO titer indicates recent infection with Streptococcus. It is advisable to administer penicillin for the infection. The arthritis can be managed with salicylates. The patient will need long-term treatment with penicillin as a result of his rheumatic fever.

A 25-year-old woman is discovered on physical examination to have a midsystolic click and a high-pitched heart murmur. Which of the following additional cardiovascular findings is she most likely to exhibit?

PVC. A midsystolic click and a high-pitched heart murmur in a young woman is the classic presentation of mitral valve prolapse. Patients are usually asymptomatic but may have dyspnea, tachycardia, chest pain, syncope, eventual congestive heart failure, or, rarely, sudden death. Prolapse may coincide with tricuspid or pulmonary valve disease or with psychiatric conditions such as anxiety or depression. Complications may include atrial thrombosis, calcification, infective endocarditis, emboli to the brain, rupture of chordae, mitral regurgitation, arrhythmias, and premature ventricular contractions (PVCs). Atrial fibrillation (choice A) may result from chronic mitral stenosis, not mitral valve prolapse. This valvular disease is associated with an early diastolic opening snap. Decreased peripheral pulse pressure (choice B) and slowed carotid upstroke (choice D) are seen in aortic valve stenosis. A systolic ejection click is associated with this valvular disease. Wide pulse pressure (choice E) is seen clinically as bounding pulses. It is associated with aortic valve insufficiency. Mitral valve prolapse is the most common valvular heart condition in the United States. Most patients are asymptomatic and rarely have cardiac-related symptoms. A conclusion from the Framingham Heart Study found that the only association found with mitral valve prolapse was low body weight. If a patient does have symptoms, palpitations are the most common manifestation.

Which of the following EKG abnormalities is treated as an equivalent to an ST-segment elevation acute myocardial infarction in a patient who has acute onset of chest pain?

Patients who have acute signs and symptoms consistent with myocardial infarction pattern who display a new-onset left bundle branch block pattern are treated equivalently to patients who have ST-segment elevation myocardial infarction until cardiac enzyme patterns are completed. Under appropriate circumstances, these patients are eligible for percutaneous cardiac intervention and thrombolytic therapy. Patients who have acute myocardial infarction often have such damage to the left ventricle that the repolarization pattern is affected, which results in a left bundle branch block pattern on the EKG. Other causes of left bundle branch block include heart failure, aortic stenosis, and left ventricular aneurysm. Conversely, Right bundle branch block pattern, primarily occurs in a patient who has increased pulmonary pressures or blockage of the right bundle of His. This is a commonly encountered EKG abnormality that is not typically pathologic in nature. It often occurs in the setting of left axis deviation (left anterior hemiblock), making this biphasicular block pattern very common.

A 60-year-old alcoholic female who appears malnourished presents to her health care provider complaining of shortness of breath and gasping for air on awakening. Cardiac examination reveals an S3 heart sound, a diastolic murmur, and jugular venous distention. Pulmonary rales and peripheral edema are evident. An echocardiogram would be expected to reveal which of the following?

Primary acquired cardiomyopathy can occur in the peripartum period or from tachycardia-induced dilated cardiomyopathy. Secondary, toxicity-related cardiomyopathy can occur as a result of alcohol, doxorubicin use, or from heavy metal or chemical exposure. Echocardiogram will show normal or decreased wall thickness, poor wall thickening in systole, left ventricular dilation in a spherical pattern, and diminished stroke volume. Patients who have this disorder tend to present with fatigue and dyspnea. Suspect this diagnosis in any alcoholic patient presenting with symptoms and signs of congestive heart failure. In this patient, an echocardiogram would be expected to reveal bilateral ventricular dilatation with impaired contraction throughout both chambers.

A 45-year-old woman comes to the emergency department complaining of recurring episodes of chest pain that each last a few minutes since she awoke this morning. She denies shortness of breath, radiation of the pain, or chest pain on exertion. She reports that she had similar episodes within the last month. The patient states that she has been healthy otherwise, takes no medications, and has no family history of coronary artery disease. Her vital signs and physical examination are within normal limits. Electrocardiogram shows transient ST-segment elevation. Serial cardiac enzymes reveal no abnormalities over a 24-hour period. Cardiac catheterization shows no significant areas of plaque or stenosis. Which of the following classes of drugs may be most effective in this patient?

Prinzmetal angina, or variant angina, is classically characterized by angina without the associated precipitating factors of angina pectoris. It is caused by a transient coronary artery spasm and displays a transient ST elevation on electrocardiogram. It usually has no other diagnostic findings. Calcium channel blockers have been shown to treat and prevent episodes of coronary vasospasm by decreasing contractility of cardiac muscle and producing vasodilation.

A woman is brought into the emergency department following an automobile accident in which her chest was hit by the steering wheel. Her blood pressure is 120/90 mm Hg. When she inhales, her systolic blood pressure drops to 100 mm Hg. This finding defines which of the following terms?

Pulsus paradoxus is defined as a fall in systolic blood pressure >10 mm Hg on inspiration. It can be associated with cardiac tamponade and chronic obstructive pulmonary disease (COPD).

A 71-year-old man comes to his health care provider for follow-up of a recent emergency department visit. The patient has a 2-year history of mild congestive heart failure in the setting of longstanding hypertension. He reports that yesterday he sought care at the local emergency department for palpitations and shortness of breath. He was told that his heart was "fibrillating," but later the fibrillation had "stopped on its own." His medications include a thiazide diuretic and an ACE inhibitor. On physical examination he appears well and in no distress. His blood pressure is 130/80 mm Hg and pulse 100/min and regular. His lungs have scant bibasilar rales and no gallops are appreciated. He has a grade 2 holosystolic murmur heard best at the apex. His jugular venous pressure (JVP) is 10 cm at 30 degrees. An electrocardiogram taken in the office reveals atrial fibrillation at a rate of 94/min with normal ST segments. Which of the following is the most appropriate next step in management?

Recall that in the treatment of medical conditions, certain medications overlap syndromes and are efficacious in many areas. This "co-treatment" option maximizes the benefits of each drug in a regimen and often addresses 2 or more issues simultaneously. Ischemia-mediated arrhthymia is the most common cause of death in a person who has congestive heart failure (CHF). Beta-blockers will control the rate in atrial fibrillation, as well as provide a mortality benefit for the CHF. Beta-blockers are both anti-ischemic and antiarrhythmic. In this case, ACE inhibitors have been shown to be very beneficial in prolonging the survival of patients who have CHF, as they are effective as vasodilators. They are also useful antihypertensive agents. Given this, discontinuing his ACE inhibitor (choice A) is clearly incorrect. This patient requires rate control for his atrial fibrillation so that acute heart failure does not recur. Short of restoring this patient's atrial contractions, rate control is the best method to ensure adequate management of atrial fibrillation. Digoxin, with or without a nodal agent such as a beta-blocker, has been shown to be reasonably effective at rate control, but it is less effective in high adrenergic states. Beta-blockers such as metoprolol (Lopressor) and the combined alpha- and beta-blocker carvedilol (Coreg), however, have been clearly shown to lower mortality with CHF, in addition to just controlling the heart rate.

A 12-year-old child is brought to his pediatrician for a routine health maintenance visit. He has been well except for occasional attacks of asthma and has met all development milestones. His immunizations are up to date. He occasionally uses albuterol for his asthma. Physical examination is remarkable for a blood pressure of 150/90 mm Hg in both arms. Which of the following is the most likely cause of his hypertension? Albuterol, Chronic Lung Disease, Coarctation of the Aorta, Congenital Heart Disease, Renal Disease

Renovascular disease is the most frequent cause of secondary hypertension in young children. Ailments such as polycystic kidney disease, congenital vascular disease, tumors, and infections can all lead to hypertension, and a urologic evaluation is imperative. Patients who present with hypertension at the extremes of age or whose hypertension is not responsive to traditional agents should have an evaluation performed to assess for secondary causes of hypertension.

A 5-year-old girl from Connecticut is brought to the family clinic with a 3-day history of fever and intermittent joint pain. She is generally healthy, but according to her mother, she had a "cold" about 1 month ago. On physical examination her temperature is 39.6°C (103.2°F), blood pressure 94/60 mm Hg, pulse 114/min, and respirations 22/min. Knees and elbow joints are swollen, warm, and tender to palpation. There is a grade III/VI diastolic murmur best heard at the apex. Multiple fine, pink macules are noted on her trunk. These macules blanch in the center. Which of the following is the most likely diagnosis?

Rheumatic fever is an inflammatory disease, possibly autoimmune in nature. Immune responses to group A streptococcal antigens during pharyngitis resulting in antibody cross-reactions with myocardial antigens remain central to the pathogenesis. Rheumatic fever involves many tissues, including the heart, joints, skin, and CNS. Preceding infection with group A Streptococcus is a prerequisite to the development of acute rheumatic fever. Initially, fever, dyspnea, chest pain, and cardiac murmur develop. Jones criteria (CANCER). Lyme disease may begin with migratory polyarthralgias and progress to attacks of asymmetric oligoarthritis in large joints. Erythema chronica migrans, the diagnostic annular red lesion at the site of the tick bite, occurs in 50 to 80% of cases during the early phase of illness. Cardiac involvement is atypical. If it does occur, it typically causes heart block.

A 1-week-old baby girl has a coarctation of the aorta just distal to the subclavian arteries. The blood pressure distal to the constriction is 50% lower than normal. Which of the following is increased in this infant?

The aorta is constricted at a point beyond the arterial branches to the head and arms but proximal to the kidneys. Collateral vessels in the body wall carry much of the blood flow to the lower body, and the arterial pressure in the lower body is about 50% lower compared with the pressure in the upper body. The lower than normal pressure at the level of the kidneys causes renin to be secreted with the eventual conversion to angiotensin I and angiotensin II. The renin-aldosterone system results in salt and water retention, so that within a few days to weeks the arterial pressure in the lower body (at the level of the kidneys) increases to normal, but in doing so, the blood pressure in the upper body has increased to hypertensive levels. This is the underlying pathophysiology as to why coarctation of the aorta is a secondary cause of hypertension. Correction of the coarctation will then allow the kidney to be normally perfused, and this will result in a "cure" for the patient's hypertension.

A 75-year-old man with a 40 pack per year history of smoking and hypercholesterolemia is diagnosed with severe atherosclerosis. Atherosclerotic occlusion of which of the following arteries would result in insufficient perfusion of the urinary bladder?

The bladder is supplied by the vesicular branches of the internal iliac arteries. The internal iliacs arise from the common iliac artery.

A 12-year-old African American boy is brought to the office for a well child examination. He has been in good health and only complains of an occasional headache. He has been doing reasonably well in school but has some social problems that his mother attributes to his physical appearance. She has tried persuading her son to eat healthier, but he seems to enjoy eating fast food much more than home cooked meals. His past medical history is unremarkable, and the family history is significant for adult-onset hypertension in his father's family. He takes no medication. On physical examination, the patient is in no acute distress. He is 145 centimeters tall and weighs 92 kilograms. His vital signs are within normal limits, but his blood pressure is 145/90 mm Hg. The health care provider checks that the cuff size is appropriate and remeasures his blood pressure in all four extremities, only to confirm that it is above the ninety-fifth percentile for his age. After discussing the finding with his mother, he is scheduled for several more blood pressure evaluations over the following 6 weeks. All readings yield results mildly above the ninety-fifth percentile for his age. Which of the following recommendations is most appropriate at this time?

The child should initiate a weight reduction diet with limited salt intake and regular exercise. Systemic hypertension is defined as blood pressure above the ninety-fifth percentile for age on repeated measurements over a 6-week period. Primary (essential) hypertension usually has no known underlying cause. Predisposing factors include heredity, salt intake, stress, and obesity. It is more commonly seen in adolescents and adults. Secondary hypertension is caused by an associated disease. It is more common in infants and younger children. The most common cause of secondary hypertension in children is renal disease (75-80%). A prior urinary tract infection is seen in 25 to 50% of cases, often related to an obstructive lesion of the urinary tract. In newborns, a history of umbilical artery catheterization may be elicited, with resultant thrombosis of the renal artery. Hypertension usually presents with no symptoms, especially in adolescents who have essential hypertension, and it is diagnosed on routine examination. Headaches, dizziness, vision changes, and seizures may be present. Blood pressure should be measured over several visits and compared with normal values for age. It should be measured in all extremities to rule out coarctation of the aorta. All children who have secondary hypertension should have a renal evaluation, including culture, ultrasound, renin levels, blood urea nitrogen, and creatinine. Echocardiography should be used to assess ventricular function and size. Therapy of hypertension in children is the same as in adults: diet, exercise, and medication. Pharmacologic management includes angiotensin-converting enzyme inhibitors, calcium-channel blockers, and diuretics.

On a routine physical examination, a midsystolic ejection murmur is detected in the pulmonic area of a 35-year-old woman. The cardiac examination also reveals a prominent right ventricular cardiac impulse and wide and fixed splitting of the second heart sound. EKG shows right axis deviation and chest radiograph shows enlargement of the right ventricle and atrium. Which of the following is the most likely diagnosis?

The classic findings in atrial septal defect are a prominent right ventricular cardiac impulse, a systolic ejection murmur heard in the pulmonic area and along the left sternal border, and fixed splitting of the second heart sound. These findings are caused by an abnormal left-to-right shunt through the defect, creating a volume overload on the right side. The increase in volume on the right side creates the flow murmur, the dilatation of the right-sided chambers, and the delayed closure of the pulmonic valve, all of which are present in this case. The delayed closure of the pulmonic valve occurs because of the increased volume of blood that is delivered to the right ventricle.

An elderly man presents with complaints of dizziness, headaches, diarrhea, nausea and vomiting, weakness, palpitations, and a change in vision with a yellowish to blue tint to his vision. He is taking multiple medications. He has a history of chronic heart failure and hypertension. His wife states that he has had a few episodes of confused, delirious behavior over the past few weeks. Which of the following agents might be responsible for this man's symptoms? Allopurinol, Hydralazine, Niacin, Digoxin, Spironolactone

The collection of symptoms described—dizziness, headaches, diarrhea, nausea and vomiting, weakness, palpitations, and a change in vision with a yellowish to blue tint to the vision—are classic side effects of digoxin (Lanoxin). EKG changes, such as biventricular tachycardia, may also occur. Digoxin is a medication that has been shown to improve symptoms in patients who have heart failure, as it is a positive inotropic and negative chronotropic agent. Patients who are hypokalemic are especially prone to the development of digoxin toxicity.

A 47-year-old woman with a history of rheumatic fever is examined by her health care provider. Physical examination is significant for a low-pitched, rumbling, diastolic murmur preceded by an opening snap. This murmur radiates into the apex of the heart. The affected valve can be best evaluated by auscultation at which of the following locations?

The correct answer is B. Solid knowledge of cardiac anatomy and its clinical correlations are crucial for performing physical examinations. This question tests two facts. First it requires that you recognize the patient has mitral stenosis. Classic clues to this diagnosis are "low-pitched, rumbling, diastolic murmur," and "opening snap." The mitral valve is the most commonly affected valve in rheumatic fever, followed by the aortic and tricuspid valves. The mitral valve is most audible over the left fifth intercostal space at the midclavicular line.

A 56-year-old smoker with no previous medical history comes to the emergency department complaining of chest pain and shortness of breath with exertion. He is admitted to the hospital for further evaluation. An exercise stress test supports the diagnosis of coronary artery disease, and the patient undergoes a heart catheterization. There is diffuse coronary artery disease but no clearly stentable lesions. The cardiologist decides that medical management of this patient's coronary disease is appropriate at this time. The patient has no allergies and no other medical conditions. Diltiazem, HCTZ, Lisinopril, Metoprolol, Nifedipine

There is clear and convincing evidence that beta-blockers are indicated in the treatment of patients who have coronary artery disease. This is partly because in addition to lowering blood pressure, beta-blockers keep the heart rate slow, which decreases strain on the heart by decreasing myocardial oxygen demand and increases myocardial perfusion. Because this patient has coronary artery disease and hypertension, the best choice for this patient is metoprolol. Diltiazem is a non-dihydropyridine CCB that also has blood pressure lowering properties together with heart rate controlling properties. This class of medications has not been shown clearly to decrease mortality in patients who have coronary artery disease, so they are not used as first-line agents. They can be considered in patients who require heart rate control but who cannot tolerate beta-blockers (e.g., asthmatics, peripheral arterial disease). Hydrochlorothiazide together with beta-blockers can be used as a first-line blood pressure medication in many patients. It is probably an underused class of medications. Hydrochlorothiazide has synergistic blood pressure lowering effects with many other classes of blood pressure medications. Because of our patient's proven coronary artery disease, a beta-blocker would be a better choice for this patient. It would be the first add-on agent if beta-blockers failed to reach blood pressure control goals. Lisinopril. ACE inhibitors are indicated as first-line blood pressure medications in patients who have diabetes, because these agents have been shown to slow the progression of diabetic nephropathy. They also have been shown to improve survival and are first-line agents in patients who have known congestive heart failure because they act as an after-load reducer. Finally, in postmyocardial infarction patients who have left ventricular damage, ACE inhibitors seem to have a beneficial effect on survival due to a lessening of the remodeling effect on the ventricle. ACE inhibitors act as vasodilators in patients who have chronic heart failure, and they are most effective in patients who have the lowest ejection fraction from their left ventricle. Nifedipine is a dihydropyridine calcium-channel blocker that acts to inhibit calcium ion influx into vascular smooth muscle and myocardium. It acts as a predominantly peripherally acting antihypertensive. It acts as a peripheral vasodilator and decreases blood pressure. It is known to have the side effect of reflex tachycardia, so it is not first-line therapy for postmyocardial infarction or angina patients.

A 25-year-old man is shot with a .22-caliber revolver. The entrance wound is in the anteromedial aspect of the upper thigh, while the exit wound is approximately 3 inches lower, in the posterolateral aspect of the thigh. He has a large, expanding hematoma in the upper inner thigh. There are no palpable pulses in the foot. The bone is intact by physical examination and x-ray film. Which of the following is the most appropriate next step in management?

There is no question that this patient's femoral vessels are injured, and the expanding hematoma plus absent pulses indicate that the femoral artery is involved (the vein may or may not be injured). Surgical exploration starts with proximal and distal control. Once the hematoma is safely entered, the extent of the injuries can be ascertained and the proper repair performed.

A 41-year-old man comes to his health care provider for a routine physical examination. He is new to this office and brings his previous medical record with him. He has no significant past medical history, but he does have a strong family history of cancer and heart disease. His father and his brother both had myocardial infarctions before age 55 years, and his sister, mother, and aunt had breast cancer. He exercises regularly and eats well, with most of his diet being low in saturated fat and cholesterol. He smokes one pack of cigarettes per week. His review of systems is unremarkable. He is very anxious and would like only minimal interventions done because of his good health. Which of the following is the most age-appropriate screening test in this patient?

The current recommendations for routine, age-appropriate screening are based in some measure on data from clinical trials. Depending on the source of the recommendations, there is considerable variability in these recommendations. One current recommendation is that at least every 5 years a random cholesterol level should be checked. Typically, total cholesterol and HDL cholesterol can be drawn as a baseline in a nonfasting state. If abnormalities are found in these measurements, a fasting blood lipid (ideally a 12-hour fast) can be performed.

A 25-year-old woman involved in an automobile accident is admitted as an emergency patient. A major artery severed in her leg caused an estimated 600 mL blood to be lost. Her blood pressure is 90/60 mm Hg. Which of the following would be expected to increase in response to hemorrhage?

The decrease in blood pressure caused by hemorrhage activates the baroreceptor reflex, which tends to increase sympathetic nerve activity and decrease parasympathetic (vagal) nerve activity. The increase in sympathetic nerve activity constricts arterioles in skeletal muscle and elsewhere in the body. Her compensatory responses include the following mechanisms: baroreceptor reflex, chemoreceptor reflex, epinephrine and norepinephrine released from the adrenal medulla, formation of angiotensin II, formation of vasopressin, and the capillary fluid shift mechanism. Activation of the renin-angiotensin system during hemorrhage also plays an important role in maintaining blood pressure. Angiotensin II increases blood pressure acutely by constricting arterioles throughout the body, and chronically by decreasing the renal excretion of both salt and water.

A 52-year-old man comes to the hospital complaining of palpitations. The patient reports that while cooking breakfast this morning, he felt his heart "racing in his chest" and was unable to catch his breath. He states that sitting down brought no relief. He called for an ambulance and he was brought to the emergency department. The man has no significant past medical history and takes no medications regularly other than ranitidine (Zantac) for occasional heartburn. On examination, the patient is quite thin but well developed and in mild distress. His pulse is 140/min and irregularly irregular. There are no murmurs, and the lung examination is clear. Which of the following findings on his echocardiogram would suggest a diagnosis of longstanding atrial fibrillation?

The most common anatomic correlate seen in patients who have AF (Framingham Heart Study) is an enlarged left atrium. Patients who have an enlarged left atrium are less likely to remain in normal sinus rhythm following cardioversion. Echocardiography is an invaluable tool for assessing cardiovascular function in both normal and disease states. In the case of atrial fibrillation (AF), a chronic course versus an acute and self-limited course portends completely different treatment strategies and long-term prognoses. The most common cause of chronic AF is valvular disease, followed by congestive heart failure (CHF). Hyperthyroidism that is not adequately treated is another cause of atrial fibrillation and should be suspected in a patient who has new-onset AF.

A 50-year-old man is admitted to the hospital after sustaining an acute myocardial infarction. Eight hours after this event, his blood pressure is 70/50 mm Hg and his pulse is 45/min. An electrocardiogram reveals sinus bradycardia rhythm. Which of the following is the most appropriate intervention? atropine, dobutamine, beta blocker, pacemaker, cardiac cath

The patient is hypotensive and bradycardic. This suggests a vagal response, and administering an agent that is vagolytic, such as atropine, is the correct treatment. Atropine is given for symptomatic bradycardia post-MI at a dosage of 0.5 mg every 5 minutes up to a total of 3 mg. As the vagus nerve is blocked, the patient's heart rate will increase.

A 45-year-old man suddenly loses consciousness and falls to the ground. His pulse is lost. He has been previously healthy and has been on no medication. There is no obvious evidence of trauma. Electrocardiogram reveals wide complex tachycardia at a rate of 200/min. CK-MB levels are elevated. Which of the following is the most appropriate intervention?

The patient is in ventricular tachycardia and is hemodynamically unstable as illustrated by the loss of consciousness. He should be emergently defibrillated with 200 joules of energy initially. In addition, ventricular tachycardia without a pulse should be treated the same way as ventricular fibrillation.

A 68-year-old man presents with complaints of chronic fatigue, exertional and nocturnal dyspnea, orthopnea, and a chronic nonproductive cough. On examination, respiratory wheezing and rhonchi are noted. Cardiac examination reveals a diminished first heart sound and an S3 gallop. The patient indicates that he was recently treated for hypertension and vasospastic angina. On the basis of his initial presentation, which of the following agents was most likely prescribed? Amlodipine, Captopril, Furosemide, Verapamil, Hydralazine

The patient is presenting with classic signs and symptoms of congestive heart failure: chronic fatigue, exertional and nocturnal dyspnea, orthopnea, a chronic nonproductive cough, respiratory wheeze and rhonchi, as well as a diminished first heart sound and an S3 gallop. Verapamil is a non-dihydropyridine calcium channel blocker used to treat both hypertension and vasospastic angina. It has, however, a strong negative inotropic effect on the heart that can cause signs and symptoms of heart failure. Furthermore, some clinical studies have shown that congestive heart failure can develop in a small percentage of individuals taking verapamil.

A 55-year-old woman with a long-standing history of atrial fibrillation secondary to mitral regurgitation comes to the emergency department with a painful right foot. The patient reports that over the past few hours her foot has become more painful and now is nearly insensate. She describes the pain as burning and states that it is not relieved by any intervention. She takes warfarin (Coumadin), atenolol (Tenormin), digoxin (Lanoxin), and aspirin. On physical examination, her pulse is irregularly irregular. Her lungs are clear, and she has a loud holosystolic murmur heard best at the apex. Her right foot is gray and cool to the touch and has poor capillary refill. Dorsalis pedis and posterior tibial pulses are absent on the right. Her prothrombin time is 14.4 seconds (normal 11-13 seconds) and her INR is 1.4 with goal of anticoagulation being 2-3. Which of the following is the most appropriate course of action?

The symptoms and signs that she is describing, particularly in the context of atrial fibrillation (AF), suggest peripheral embolization, which is a surgical emergency. The treatment of choice involves immediate embolectomy performed by a vascular surgeon, TPA infusion therapy, or insertion of a catheter with urokinase-directed therapy. After successful embolism removal or dissolution, anticoagulation is performed along with administration of heparin, which is given to prevent new clot formation. Her subtherapeutic prothrombin time and persistent AF on examination are supportive of this diagnosis.

A 55-year-old man with a recent syncopal episode is admitted to the hospital with congestive heart failure. His blood pressure is 160/100 mm Hg and pulse 90/min. He has a grade 2/6 harsh systolic ejection murmur. An echocardiogram reveals a thickened ventricular septum and systolic anterior motion of the mitral valve. Which of the following will most likely be found in this patient?

The thickened ventricular septum and the systolic anterior motion of the mitral valve suggest hypertrophic cardiomyopathy. The murmur is harsh and systolic and decreases when afterload increases as a result of hand grip exercise. Hand grip increases systemic vascular resistance, which allows the heart to fill with blood against increased afterload; this filled ventricle will lessen the obstruction that occurs with hypertrophic cardiomyopathy. The Valsalva maneuver decreases venous return of blood to the heart. This results in less blood in the heart, which decreases the left ventricular cavity size and worsens the obstruction in hypertrophic cardiomyopathy. Squatting increases both venous return and chamber size. It also increases systemic arterial resistance and increases most murmurs, except those caused by hypertrophic cardiomyopathy. The increase in the chamber size from the increased venous return of blood to the heart during squatting causes a reduction in the murmur.

A 15-month-old boy is brought to the emergency department by his parents because he suddenly turned blue and had difficulty breathing while he was playing in the back yard of their home. The parents have noticed several times in the previous 2-3 months that he had developed a bluish discoloration around the lips but did not think much of it, as it was wintertime. Recently, however, the boy had increasing fatigability and would stop in the middle of playing to catch his breath by sitting down or squatting. This morning when he suddenly turned blue and started behaving in a very flustered manner they rushed him to the emergency department. The parents deny any other significant medical conditions or allergies to medication. They have a 4-year-old daughter who is in good health. On physical examination, the patient is in the fifth percentile for height and weight. His lips and fingertips are bluish in color and he frequently stops to catch his breath while playing in the examination room. There is a hint of clubbing of his fingers. Vital signs are within normal limits, but a complete blood count shows a red blood cell count of 6 x 1012/L and a hematocrit of 66%. A chest radiograph shows a boot-shaped heart with an uptilted apex and clear lung fields. On auscultation, there is a harsh systolic ejection murmur and a single S2 is heard. Which of the following is the most likely diagnosis?

This boy has a congenital heart defect known as tetralogy of Fallot. Tetralogy of Fallot is defined as pulmonary stenosis, ventricular septal defect, dextroposition of the aorta (overriding), and right ventricular hypertrophy. It is the most common type of cyanotic congenital heart disease. Symptoms depend on the size of the ventricular septal defect and the degree of the right ventricular outflow tract obstruction. Acyanotic (pink) tetralogy occurs when there is sufficient pulmonary blood flow caused by mild obstruction and the shunting across the ventricular septal defect is balanced. Typically, however, patients present with cyanosis, delayed growth and development, and dyspnea. Paroxysmal hypercyanotic attacks (hypoxic, blue, or tet spells) manifest with episodes of restlessness, cyanosis, and gasping respirations. Clubbing of the fingers and toes occurs secondary to chronic hypoxia. A loud, harsh, systolic ejection murmur is heard. S2 is single or very soft because of the pulmonary stenosis. Chest radiographs reveal a boot-shaped heart (coeur en sabot) with uptilted apex. Lung fields are clear, reflecting decreased pulmonary blood flow. Electrocardiography shows right ventricular hypertrophy and right-axis deviation. An echocardiogram reveals the anatomic abnormalities. Medical management includes maintaining the ductus open in severe right-sided obstructive lesions, but surgical correction is the definitive treatment. Blue spells are treated by placing the child in knee-chest position, sedation, oxygen, and avoiding acidosis. Beta blockade with propranolol, appropriate fluid status, and maintaining the hematocrit at 55 to 65% are also helpful. Complications have become less frequent as the surgical correction is typically performed early. Cerebral thrombosis occurs with extreme polycythemia and dehydration. Patients are usually younger than 2 years of age. Brain abscess, although less common than thrombosis, is more often seen in patients older than 2 years of age. Tet patients are at higher risk for bacterial endocarditis.

A 5-month-old girl is brought to the office by her mother, who states that the girl had an episode following feeding during which she began to breathe deeply, became blue, and then lost consciousness. The mother states that she picked her up and held her, and the infant regained her usual color and became alert. Physical examination reveals a harsh systolic murmur. The remainder of the physical examination is unremarkable. Which of the following is the most likely diagnosis?

This infant is experiencing a hypoxemic spell, as seen in patients who have tetralogy of Fallot. These hypercyanotic spells, or "Tet spells," usually are self-limited and last less than 10-15 minutes. The spells often occur immediately after feeding or when the child is crying vigorously. Tetralogy of Fallot has the following components: (1) ventricular septal defect, (2) overriding aorta, (3) right ventricular hypertrophy, and (4) pulmonic stenosis.

A 60-year-old man comes to the emergency department complaining of tearing chest pain that began abruptly and radiates to the back. His electrocardiogram is normal. Chest radiograph shows a widening of the mediastinum. What is the most likely diagnosis?

This is a classic presentation of aortic dissection. The patient may be in shock, and his pulses may be unequal. Usually there is a history of hypertension or Marfan syndrome in the affected patient. Aortic dissection is a potentially lethal cardiac emergency and requires immediate medical or surgical intervention. If it is misdiagnosed or left untreated, it may lead to mortality. Aortic dissection is classically described as a tearing sensation in the chest. This condition results from a tear in the intimal wall of the aorta. This results in blood flow being diverted into a false chamber. Chest film will show a widened mediastinum. MRI scan, CT scan, or echocardiogram are used in this evaluation. Treatment of this condition is primarily based on the location of the dissection, with proximal dissections being treated with surgical intervention and distal dissections being treated medically with blood pressure management. Blood pressure control in these patients is an essential part of treatment of this condition.

A 4-year-old, apparently healthy child is examined by a health care provider. The health care provider hears a loud systolic ejection murmur with a prominent systolic ejection click. He also hears a soft, early diastolic murmur. Both murmurs are heard best at the upper right sternal border. An electrocardiogram shows left ventricular hypertrophy. The child is active and participates in normal activities without problems. Which of the following is the most likely diagnosis?

This is aortic valve stenosis, which accounts for 5% of diagnosed cardiac defects but may actually be the most common congenital anomaly of the heart, because many minor cases are never diagnosed. Most cases are caused by bicuspid aortic valves and characteristically produce a systolic ejection murmur. An accompanying aortic insufficiency may produce an early diastolic murmur. The timing of surgical correction depends on the severity of the individual case.

A health care provider examines a 2-month-old infant who had been born at term. There is a continuous murmur at the upper left sternal border. The murmur radiates over the lung fields anteriorly. The peripheral pulses in all extremities are full and show widened pulse pressure. Which of the following is the most likely diagnosis?

This is patent ductus arteriosus, which is a failure of closure of the duct between the pulmonary artery and the aorta. As many as 80% of significantly premature (<28-week gestation) infants have patent ductus arteriosus. In term infants, delayed closure is diagnosed if the murmur of the patent ductus (described in the question stem) is still present at 6-8 weeks of age. Indomethacin is given soon after birth in premature infants and is typically started within the first 12 hours of life. This treatment is given only during the first 3 days of life in an attempt to close the patent ductus. Infants should be evaluated for other cardiac disease, as a patent ductus arteriosus may be partially compensating for other cardiac anomalies. Infants who have heart failure require prompt surgical correction. Infants who do not have heart failure or complicating cardiac defects typically undergo elective surgery at 6 months to 3 years of age to reduce the risk for infective endocarditis later involving the patent ductus. If this condition is identified in the adult population, treatment consists of a percutaneous catheter to close or surgical ligation of this structure. Coarctation of the aorta will cause diminished and delayed pulses in the legs and sometimes the arms. There may be a differential in blood pressure readings between the arms. There is typically higher blood pressure readings in the upper extremities compared to the lower extremities.

A 50-year-old surgical nurse consults a health care provider because of a rash above both her ankles. She says that her ankles have been swollen for a few months but she only recently noticed the rash. Physical examination demonstrates marked ankle edema with erythema, mild scaling, and brown discoloration of the overlying skin of the distal lower legs. Varicose veins are also noted. Which of the following is the most likely diagnosis?

This is stasis dermatitis, which is a persistent inflammation of the skin of the lower legs. The condition is often related to varicose veins, although it has been postulated that the true cause may instead be perivascular fibrin deposition and abnormal small vessel vasoconstrictive reflexes. This presentation is typical. Most patients are relatively asymptomatic and may not seek medical attention until the edema becomes severe or the lesions become complicated by secondary bacterial infection or ulceration. It is important to increase the venous return to the heart by elevating the ankles while resting and using properly fitted support hose. Unna boot application can be helpful. Purulent lesions can be treated with hydrocolloid dressings. Ulcers are treated with compresses and bland dressings, such as zinc oxide paste.

When intravenous lines cannot be established, the preferred alternate route is intraosseous (IO) cannulation. This is done by placing a trocar in the bone marrow of a long bone. The site of choice in children is the proximal tibia; alternative sites are the distal tibia and proximal femur. Careful attention should be made in regard to IO location in order to avoid injury to the growth plate which could stunt bony growth as the child grows older.

This is tetralogy of Fallot, in which severe obstruction of right ventricular outflow and a ventricular septal defect allow unoxygenated blood to pass from the right side of the heart to the left. In severe cases, cyanosis presents at birth; in milder cases (such as this infant has), it develops more slowly. The upper left sternal border ejection murmur is caused by right ventricle outflow obstruction. The ECG and chest radiographic findings described in the question stem are typical for older infants who have tetralogy of Fallot. Early surgical repair is now recommended for tetralogy of Fallot.

A 4,000-g male neonate develops severe cyanosis that begins within minutes of birth. Blood drawn 1 hour after birth shows metabolic acidosis with respiratory acidosis. A chest radiograph shows a narrow mediastinum, narrow heart base, and absence of the pulmonary artery. An electrocardiogram is normal. An echocardiogram is ordered and the report is pending. Which of the following is the most likely diagnosis?

This is transposition of the great arteries, in which the aorta arises from the right ventricle and the pulmonary artery arises from the left ventricle. Approximately 5% of congenital cardiac anomalies have transposition of the great arteries. Affected infants present within minutes of birth with severe cyanosis and metabolic acidosis secondary to inability to oxygenate tissues. The only exchange of blood between pulmonic and circulatory systems is typically occurring through a patent ductus arteriosus. The chest radiograph changes include a narrow mediastinum, narrow heart base, and absence of the pulmonary artery; these are caused by superposition of the great vessels (rather than the normal side-to-side position). Surgical repair is usually performed within 7 to 10 days of life. In transposition of great vessels, because the aorta is coming out of the right ventricle it will completely cover the pulmonary trunk. This will be seen as absence of the pulmonary artery with a narrow mediastinum on chest radiograph ("egg on a string" appearance). With this transposition, the pulmonary artery comes out of the left ventricle. Because of these changes, deoxygenated systemic venous blood travels from the right ventricle to the aorta without passing through the lungs, and pulmonary venous blood travels from the left ventricle to the lungs via the pulmonary artery. The systemic and pulmonary circulations parallel each other rather than occur sequentially. This is not compatible with life unless the two chambers connect via VSD, ASD, or patent ductus arteriosus. Prostaglandin E1 is used to maintain patency of the ductus arteriosus. An arterial switch operation is performed in the newborn period most commonly

A 71-year-old white man comes to the emergency department because of blurry vision and blood-tinged urine. He states that he has a long history of hypertension treated with a beta-blocker, an ACE inhibitor, and a calcium channel blocker. He reports that he has had 3 days of blurry vision and urine that is "cola" colored. The symptoms began when he ran out of his medications 3 days ago and he has "not had the time" to get the prescriptions refilled. His physical examination is remarkable for a blood pressure of 200/110 mm Hg in both arms and funduscopic examination showing flame hemorrhages and papilledema. Urinalysis shows red blood cells and some dysmorphic red blood cell casts. He has an abdominal bruit. Which of the following is the most appropriate next step in management?

This patient has a hypertensive emergency, as evidenced by both the presence of elevated systolic and diastolic blood pressure and evidence of end-organ damage. The blood pressure numbers themselves are a good reason for urgency, but the presence of increased intracranial pressure and renal failure require that this patient be triaged as an emergency. Elevated blood pressure alone, in the absence of symptoms or end-organ damage, rarely requires emergency therapy. The therapy for a hypertensive emergency requires IMMEDIATE LOWERING of the blood pressure by approximately 20-30 mm Hg by intravenous administration of medication. The blood pressure should not be reduced to normotensive levels because of the risk for watershed cerebral infarcts with such a dramatic reduction.

A 22-year-old man comes to the emergency department with a 3-day history of fever, chills, a cough, pleuritic chest pain, and low back pain. He says that the symptoms came on "out of the blue." He is the son of a wealthy local businesswoman and still lives at home, which he says "is cool because my parents are never around." His temperature is 39°C (102.2°F), blood pressure 120/80 mm Hg, pulse 70/min, and respirations 16/min. Physical examination shows oval retinal hemorrhages with a clear, pale center and pinpoint lesions between his toes. Blood cultures are drawn. A chest radiograph shows multiple patchy infiltrates. Laboratory studies show hemoglobin 11 g/dL, hematocrit 39%, and erythrocyte sedimentation rate 39 mm/h. Which of the following is the most likely pathogen?

This patient has acute bacterial endocarditis, most likely caused by Staphylococcus aureus, the most common organism causing endocarditis in intravenous drug abusers. The "pinpoint lesions" between his toes are signs of injection drug abuse. Acute endocarditis in drug abusers typically presents with a high fever, pleuritic chest pain, and a cough. The tricuspid valve is commonly affected in these patients. A murmur may not be present in early acute endocarditis or in injection drug abusers who have tricuspid valve disease. Roth spots are oval, pale, retinal lesions that are surrounded by hemorrhages. Other physical manifestations of endocarditis include Janeway lesions, which are hemorrhagic, painless, macular plaques typically located on the palms and soles, and Osler nodes, which are small, painful nodular lesions typically found on the pads of the fingers or toes. Other findings include anemia and an elevated erythrocyte sedimentation rate. Diagnosis is with blood cultures, which are typically positive for S. aureus, and with echocardiography.

The EKG of a 60-year-old man reveals widened QRS intervals of 0.14 seconds with distinctly abnormal configurations. Physical examination is significant for paradoxic splitting of the second heart sound. Which of the following valvular defects is likely in this patient?

This patient has bundle branch block, as implied by the QRS interval greater than 0.12 seconds and by paradoxic splitting of the second heart sound. The typical sequence of valve closure is mitral, tricuspid, aortic, and pulmonic. If there is a paradoxic splitting of the second heart sound, the aortic valve closes after the pulmonic valve. This situation can occur with anything that slows the conduction in the left ventricle, such as complete heart block, left bundle branch block, or aortic stenosis. Left bundle branch block results in conduction occurring in a cell-to-cell depolarization pattern that slows the conduction through the left ventricle, which causes the aortic valve to close after the pulmonic valve.

A 10-year-old girl is brought to the office because of fever and chills for 3 days. She had been complaining of a headache and feeling tired for 10 days before she developed fever. The parents had tried giving her acetaminophen, but the child's condition was not improving. Her past medical history is significant for frequent streptococcal throat infections over the past 2 years and a new onset heart murmur detected at her last well child visit 2 months earlier. She is on no medication currently, except for acetaminophen. Otherwise, she has been wearing dental braces for the past year and had a primary tooth extracted 2 weeks earlier in the attempt of liberating space for permanent teeth. On physical examination, the patient is diaphoretic, in moderate distress with a temperature of 39.0°C (102.2°F), blood pressure of 90/60 mm Hg, pulse of 110/min, and respirations of 24/min. On her fingernails, you note several splinter hemorrhages. Auscultation confirms a grade 2/6 high-pitched, blowing, systolic ejection murmur, best heard at the apex that radiates to the left axilla. A chest radiograph is unremarkable, but the electrocardiogram shows signs of left ventricular strain. An echocardiogram reveals vegetations on the mitral valve. Which of the following microorganisms is the most likely cause of this patient's current condition?

This patient has developed acute endocarditis most likely caused by Streptococcus viridans that entered the bloodstream during the recent dental procedure. The predisposing factor for her current illness is rheumatic heart disease, a sequela of prior Group A β-hemolytic streptococcal pharyngitis, which was complicated by involvement of the mitral valve. Endocarditis is most commonly caused by S. viridans, although in some series S. aureus is more common, especially when there is no underlying heart disease. S. viridans is more common if endocarditis occurs after dental procedures. P. aeruginosa and S. marcescens are seen as common pathogens in intravenous drug abusers. Fungal causes are seen after open-heart surgery. Endocarditis is frequently associated with congenital or rheumatic heart disease. Surgical or dental procedures are commonly the predisposing incident. The highest risk is seen in patients who have high-velocity blood flow, such as in ventricular septal defect and left-sided obstructive lesions. Clinically, the onset of symptoms may be acute or insidious. Fever, chills, arthralgias, myalgias, development of a new murmur, splenomegaly, and petechiae may occur. S. aureus infection is associated with neurologic complications. Skin manifestations are seen late in the disease evolution and most likely represent vasculitis. These include Osler nodes (tender nodules on the finger and toe pads), Janeway lesions (painless hemorrhagic lesions on the palms and soles), and splinter hemorrhages (linear lesions beneath the nails). A positive blood culture is the most important piece of information in establishing the diagnosis of endocarditis. Leukocytosis, increased sedimentation rate, and anemia are helpful but secondary in importance to a positive blood culture. Echocardiography may reveal vegetations on the heart valves. Antibiotics are the mainstay of treatment of endocarditis. Therapy should last 4 to 6 weeks. Congestive heart failure should be treated if present prevention is with antibiotic prophylaxis before and after dental or surgical procedures. Complications of endocarditis include heart failure and systemic embolism. Mortality rate is 20 to 25%.

A 47-year-old man comes to his primary care provider for follow-up care of his hypertension. The patient has a 10-year history of hypertension that has been treated with a variety of medications. It is now well controlled with a beta-blocker and thiazide diuretic combination. The patient's mean blood pressure over the past 6 months has been 145/85 mm Hg. His other medical history is remarkable only for type 2 diabetes mellitus. He currently takes a low-dose oral sulfonylurea agent and has had blood glucose values ranging from 110-220 mg/dL. His most recent hemoglobin A1c value was 7.9%. He has no allergies to any medications and he quit smoking cigarettes 2 years ago. Which of the following is the most appropriate management at this time?

This patient has diabetes and hypertension. The management strategy for such patients includes addressing the primary blood sugar problem, the primary blood pressure problem, and the secondary consequences of both the diabetes and the blood pressure. This man has not reached the blood pressure goals for patients who have diabetes, which is 130/80 mm Hg. To further lower his blood pressure, an appropriate intervention would be to add an ACE inhibitor to deal with diabetic complications, such as renal disease, as well as to address further lowering of his blood pressure. Increasing his sulfonylurea dose to better control his sugar is also appropriate, because the patient's hemoglobin A1c is not at the goal of <7%.

A 61-year-old woman comes to the health care provider for her first physical examination in more than 10 years. She reports that she has been in excellent health, does not smoke or drink, and runs 3 miles daily. She is a retired accountant and has 3 healthy grown children. She has been taking 81 mg of aspirin daily after reading about its cardioprotective effects in the newspaper. On physical examination she appears well. Blood pressure is 122/76 mm Hg, pulse 70/min, and respirations 14/min. She is afebrile. Head and neck examination is normal. There is no jugulovenous distention. Lungs are clear. On cardiac examination she has a regular S1 and S2, and II/VI crescendo blowing diastolic murmur is heard at the aortic area. Abdominal examination is normal. Rectal examination shows no masses and brown, guaiac-negative stool. Which of the following most likely explains the cardiac findings on physical examination?

This patient has findings of asymptomatic aortic insufficiency on physical examination. This lesion may result from a number of causes, several of which are infectious in etiology. Aortic insufficiency may result as a sequela of rheumatic heart disease, which occurs as an immunologic response to a streptococcal infection. Acute rheumatic fever is typically characterized by cardiac involvement that may cause pericarditis, myocarditis, or endocarditis. Often the initial cardiac manifestations are asymptomatic and become apparent only years later with the development of cardiac valvular disease.

A 25-year-old man comes to the health care provider with chest pain at rest that is not always related to exercise. He reports two prior episodes of fainting during exercise but has otherwise been healthy. He reports a similar history in other family members and notes that his father suddenly collapsed and died at age 50 after playing tennis. He denies use of cocaine or other recreational drugs and does not take any medications. Physical examination reveals a systolic ejection murmur that is loudest along the left sternal border. The rest of the physical examination is unremarkable. Echocardiography shows asymmetric septal hypertrophy without obstruction. Which of the following interventions would be most likely to decrease this patient's systolic murmur?

This patient has hypertrophic obstructive cardiomyopathy. This is a cardiac condition in which the interventricular septum hypertrophies excessively and asymmetrically. The hypertrophied septum and the anterior leaflet of the mitral valve produce left ventricular outflow obstruction. Most cases are inherited through an autosomal-dominant mode of transmission. The systolic ejection murmur is diminished when the patient lies down, because this maneuver increases cardiac size by increasing venous return and tends to diminish the intensity of the murmur. Afterload is increased, and venous return is increased to the heart. This increases the ventricular size and diminishes the murmur. Maneuvers such as Valsalva which diminish left ventricular size bring the anterior leaflet of the mitral valve closer to the septum and increase the amount of obstruction. This increases the intensity of the murmur.

A 57-year-old woman with a history of rheumatic fever as a child comes to the her health care provider complaining of a 6-month history of slowly progressive dyspnea on exertion and orthopnea. Her temperature is 37°C (98.6°F), blood pressure is 110/60 mm Hg, pulse is 93/min and irregular, and respirations are 18/min. Cardiac examination reveals a localized mid-diastolic murmur near the apex. There is a loud opening snap heard after S2. The rhythm appears irregular. Which of the following additional findings will most likely be present on physical examination?

This patient has mitral stenosis. Most adults who have mitral stenosis have had rheumatic fever as a child, although not all patients are aware of having had this infection. Mitral stenosis decreases left ventricular filling and elevates left-sided atrial pressures. This causes pulmonary congestion and results in symptoms of left-sided heart failure, such as shortness of breath and dyspnea on exertion. Hemoptysis sometimes occurs as a result of rupture of small pulmonary blood vessels. Later in the course of the disease, patients may develop pulmonary hypertension and cor pulmonale because the right ventricle has to work against increased pressures secondary to chronic pulmonary congestion. There may be a loud P2 as a sign of pulmonary hypertension and/or right ventricular failure. Patients may have an opening snap caused by the calcification of the stenotic mitral valve. Mitral stenosis is a harsh mid-diastolic murmur heard best at the apex of the heart. Shortening of the interval before the opening snap indicates a higher degree of stenosis (the pressures in the left atrium force the valve open earlier). Atrial fibrillation is common with mitral stenosis. Early in the course of the disease, the intensity of S1 is increased. This is because the stenosis prevents the valve from closing spontaneously after diastole. Late in the course of the disease, the valve becomes too stenotic to open or close, and therefore the intensity of S1 decreases.

A 50-year-old man is brought to the emergency department complaining of light-headedness. He has a history of lung cancer, which was diagnosed a month ago and found to be widely metastatic to the bone and pericardium. On physical examination his blood pressure is 70/40 mm Hg and pulse 100/min. Heart sounds are distant and soft. ECG demonstrates low voltage, and electrical alternans is present. Chest radiograph shows that the cardiac silhouette has a "water bottle" appearance. Which of the following is the most appropriate intervention?

This patient has pericardial/cardiac tamponade, most likely as a result of his malignancy. Lung cancer is particularly likely to cause pericardial effusions. Furthermore, because this patient has metastases to the pericardium, he might be bleeding into the pericardial space. This tamponade may be the cause of his significant hypotension and the soft cardiac sounds. Electrical alternans, a phenomenon in which the QRS changes axis, is indicative of pericardial effusion, because the heart is moving freely in the fluid, causing this change in axis noted on the ECG. Emergently, this patient needs decompression of the pericardial space with the aid of pericardiocentesis, in which a catheter directly drains the fluid in the pericardial sac. Nonsteroidal anti-inflammatory drugs (NSAIDs) can be useful in treating pericarditis, which may lead to pericardial effusions. This is a longer term option, however, and will have little utility emergently. Steroids may similarly be used in pericarditis, after NSAIDs have failed. This is an option to be explored after the pericardial fluid has been drained, however.

A 67-year-old woman comes to the clinic for review of her medications. She has had a history of hypertension for 25 years, type 2 diabetes for 20 years, and congestive heart failure for 5 years. She is a former smoker of two packs of cigarettes per day and her lipid status is not known at this time. Her current medications include nifedipine (Adalat, Procardia), hydralazine (Apresoline), isosorbide dinitrate (Isordil), glyburide (DiaBeta, Micronase), a multivitamin, and conjugated estrogens. Today in the clinic her blood pressure is 160/90 mm Hg, her fasting blood glucose is 210 mg/dL, and her hemoglobin A1c is 7.9%. She reports moderate dyspnea on exertion, unchanged from previous visits. Which of the following is the most appropriate intervention at this time?

This patient has poorly controlled hypertension and poorly controlled diabetes. She needs improved therapy for both; the issue is how best to do that. The concept underlying this question is the absolute importance of using ACE inhibitor therapy on both type 1 and type 2 diabetic patients. Many clinical trials have shown the beneficial effects of ACE inhibitors on preventing nephropathy and slowing the progression of established nephropathy in diabetics. It is the standard of care that all diabetics be given an ACE inhibitor if they are able to tolerate its blood pressure effects. Given that she has congestive heart failure and hypertension, the ACE inhibitor also will be efficacious in their treatment. In fact, ACE inhibitors have been shown to be superior to hydralazine and isosorbide dinitrate in terms of morbidity and mortality in treatment of CHF; all patients who have symptomatic CHF, regardless of ejection fraction, should be placed on one.

A 72-year-old man is scheduled to have elective sigmoid resection for diverticular disease. He has a history of heart disease and had a documented myocardial infarction 2 years ago. He currently does not have angina, but he lives a sedentary life because "he gets out of breath" if he exerts himself. Physical examination reveals jugular venous distention. His hemoglobin level is 9 g/dL (normal 14-17 g/dL in men). If surgery is indeed needed, which of the following should most likely be done prior to the operation?

Treat the patient for CHF. Jugular venous distention in this setting is indicative of congestive heart failure, a condition that would make elective surgery very risky. Medical treatment for congestive heart failure can reduce the risk. This patient has right-sided heart failure, which is most commonly caused by left-sided heart failure.

A 16-year-old boy is brought to the emergency department by his parents because of an episode of severe chest pain several hours earlier. Review of symptoms reveals that the patient is recovering from a flu-like illness that started a week earlier. He had been complaining of fever, chills, abdominal discomfort, and feeling tired during the prior week. He has had a low-grade fever for 5 to 6 days and a sore throat, but pharyngeal swab cultures done at the primary care physician's office had been negative and he was taking over-the-counter cold medication only. Physical examination reveals a well developed and well nourished young man in moderate distress. His temperature is 37.8°C (100.0°F), pulse is 120/min, and respirations are 28/min. A differential white blood cell count shows a normal number of neutrophils and marginally elevated lymphocytes. An electrocardiogram shows low voltage QRS complexes throughout the limb leads. Chest radiography is remarkable for increased pulmonary markings and an enlarged heart silhouette. Which of the following is the most appropriate next step in the management?

This patient has signs of myocarditis and should be admitted to a monitored hospital bed for further evaluation and management. Myocarditis is an inflammation of the myocardium. The etiology may be multiple, but it is most commonly viral, caused by adenovirus and Coxsackie B. Bacteria (diphtheria), Rickettsia, fungi, and parasites are also infectious causes of myocarditis. Connective tissue diseases, granulomatous diseases, and toxins may cause noninfectious myocarditis. The most common clinical presentation is heart failure.

A 77-year-old man comes to the health care provider because of decreasing exercise tolerance. Just 1 year earlier he was able to play doubles tennis for 2 hours. Over the past few months, however, he has had progressive dyspnea on exertion and now can walk only 2 blocks on level ground before becoming short-winded. He has also been awaking from sleep with shortness of breath and requires 3 pillows to sleep comfortably. He has a history of rheumatic fever as a teenager. On physical examination his blood pressure is 168/60 mm Hg, pulse 92/min, and respirations 18/min. He is afebrile. He has jugulovenous distention lying supine. He has bibasilar rales extending 1/4 up both posterior lung fields. He has a regular S1 and S2, with a blowing diastolic murmur heard at the aortic area, which is grade II/VI. An S3 is audible. The liver edge is mildly tender, and there is moderate lower extremity edema extending to both knees. Which of the following medications will most likely be effective in the management of his cardiac disorder?

This patient has the physical findings of aortic insufficiency and the development of congestive heart failure with biventricular failure. Left ventricular symptoms described here are the findings of pulmonary congestion, and right-sided heart failure is demonstrated by the jugulovenous distention, congested liver, and peripheral edema. Because the "backward" failure symptoms are caused by regurgitant flow across the incompetent aortic valve, the most useful therapy would be afterload reduction with an ACE inhibitor such as captopril, which will also help to prevent cardiac remodeling as the left ventricle becomes overloaded from the increased amount (volume) of blood that occurs because of aortic regurgitation. Aortic regurgitation may be helped by vasodilator therapy, which will act as afterload reducers. In addition to ACE inhibitors, pure vasodilators such as hydralazine and nifedipine can also be used. For patients who have ongoing symptoms who are surgical candidates, aortic valve replacement is performed when the disease decompensates.

A 67-year-old man comes to the health care provider complaining of increasing dyspnea on exertion. Over the past 3 weeks, he has noted increasing shortness of breath while walking to the bus stop three blocks from his home. He has also found it difficult to sleep comfortably while lying flat and now sleeps on three pillows. He has awoken several times in the past month with shortness of breath. He denies any history of chest pain. He does not smoke, and his cholesterol tests were normal 1 year earlier. On physical examination, he appears comfortable at rest. His blood pressure is 158/56 mm Hg, pulse is 86/min and regular, and respirations are 16/min. He has jugulovenous distention while being examined at 30 degrees. His lungs have bibasilar rales. On cardiac examination, there is a regular S1 and S2, with a blowing diastolic murmur heard loudest at the left sternal border. A I/VI systolic murmur is heard throughout the precordium. An extra heart sound immediately following the S2 is heard. There is mild lower extremity pedal edema bilaterally. Which of the following is the most likely cause of his symptoms?

This patient has the typical symptoms of congestive heart failure with left ventricular failure. The physical examination has the characteristic findings for aortic insufficiency, including wide pulse pressure, blowing diastolic murmur, and an S3 consistent with left ventricular dilatation. The soft systolic murmur is caused by the volume overload and increased flow in the left ventricle and does not imply an additional valvular disorder.

A 55-year-old man is brought to the emergency department because of increasing incoherence over the past 24 hours. He has a history of hypertension and diabetes. He is disoriented. On physical examination, he is afebrile. His blood pressure is 230/130 mm Hg, pulse is 120/min, and respirations are 24/min. He has an S4 on cardiac examination. The patient is placed on a cardiac monitor, and IV and intra-arterial lines are placed. A head CT scan shows no mass or bleed. Which of the following is the most appropriate next step in management?

This patient is having a hypertensive emergency, given the presence of end-organ damage (mental status changes) in the setting of hypertension. Hypertensive encephalopathy allows the definition of hypertensive emergency rather than hypertensive urgency to be used. Immediate therapy is needed, before the laboratory results are known. This requires immediate but not precipitous lowering of the blood pressure over a period of minutes to hours. Labetolol acts as a combined alpha- and beta-blocker and will decrease systemic vascular resistance, mean arterial pressure, and heart rate with minimal change in cardiac output. It is given intravenously with an onset of action in 5 to 10 minutes with a duration of action of 3 to 8 hours. Hypertensive emergency is defined as a systolic blood pressure >210 mm Hg and diastolic blood pressure >130 mm Hg along with changes consistent with end-organ damage. When this condition is suspected, treatment should be performed without delay. Blood pressure should be lowered within minutes to hours with parenteral agents used in a monitored setting. The initial goal of therapy is to lower mean arterial blood pressure by no more than 25% within the first hour. Further blood pressure lowering may be needed in patients who have intracranial hemorrhage or aortic dissection. If the patient's blood pressure is lowered too rapidly there may be an increase in renal, cerebral, or coronary ischemia.

A 49-year-old man with a history of hypertension and hyperlipidemia has had a severe headache, blurry vision, and red urine for the past 2 days. He was in his usual state of health until 2 days ago. He has been compliant with his medications, including atenolol 50 mg/day and hydrochlorothiazide 12.5 mg/day. His temperature is 37.0°C (98.6°F), blood pressure 196/140 mm Hg, pulse 83/min, and respirations 10/min. The remainder of his physical examination is unremarkable. Which of the following is the most appropriate intervention at this time?

This patient is having a hypertensive emergency, which is defined as severe hypertension with end-organ damage. It happens for unknown reasons in approximately 1% of hypertensive patients. Our patient has hematuria, headache, and blurred vision. He needs to be admitted to the hospital and placed on intravenous blood pressure medications such as labetolol as soon as possible. Remember not to lower the blood pressure too far, too rapidly, because doing so might compromise cerebral, renal, or coronary perfusion. The goal for decreasing blood pressure is no more than 25% of the mean arterial blood pressure within the first minutes to hours.

A 57-year-old man with hypertension and hyperlipidemia comes to the emergency department complaining of crushing substernal chest pain, diaphoresis, and nausea for 3 hours. He denies palpitations or shortness of breath. He has no previous history of angina or myocardial infarction. His temperature is 37.0°C (98.6°F), blood pressure is 82/60 mm Hg, pulse is 103/min, and respirations are 20/min. Physical examination shows jugular venous distention, a tachycardic heart that is without murmurs, and clear lungs. An electrocardiogram is shown. A right-sided electrocardiogram is also performed that shows ST elevation in V3 and V4. Which of the following is the most appropriate medical intervention at this time?

This patient is having a right ventricular infarct. The triad of jugular venous distention, hypotension, and clear lungs should make you suspect the diagnosis. In addition, the electrocardiogram shows inferior ST-segment elevation (II, III, and aVF), which is consistent with RV infarct. Right-sided electrocardiogram can be used to support this diagnosis. The finding of elevations in lead V4 on a right-sided electrocardiogram is 100% specific for a right-sided infarct. The management of right ventricular infarct requires aggressive fluid resuscitation, as this type of myocardial infarction is fluid-dependent. The infarcted patient is now preload-dependent. They require aggressive intervention to keep the preload high. Aggressive intravenous fluids therefore should be started until the patient stabilizes or goes for cardiac catheterization. Intravenous fluids should be first-line in maintaining adequate blood pressure.

A 34-year-old woman with HIV and active Pneumocystis pneumonia is admitted to the hospital. Her last CD4 count was 44 cells/mm3 (normal 400-1,400 cells/mm3). She has been doing reasonably well since admission with a stable course on appropriate antibiotics. On day 2 of admission, vital signs reveal blood pressure 80/40 mm Hg and pulse 110/min. Temperature is 38.3°C (101°F) orally. Her extremities are cool and damp. Mental status examination is normal. The remainder of her physical examination is unchanged. What is the next best step in management?

This patient is hypotensive but has minimal effects from the hypotension, such as altered mental status or signs of shock. This blood pressure, however, is still not acceptable, and the cause for it must be determined. In the interim, the symptom (low blood pressure) must be treated. The standard therapy in all such situations is volume. Because she has no signs of altered mental status, there is no urgent indication for pharmacologic blood pressure support, and IV fluids are the first-line initial management tools for this patient.

A 12-year-old girl presents to her health care provider with an erythematous macular skin rash in a bathing suit distribution. She has a history of streptococcal sore throat a few weeks ago. Which of the following signs would be necessary to make the diagnosis of rheumatic fever?

This patient's macular skin rash in a bathing suit distribution is also known as erythema marginatum, one of the five major Jones criteria for diagnosing rheumatic fever. The other four major criteria are migratory polyarthritis, Sydenham chorea, subcutaneous nodules, and pancarditis. In context of prior streptococcal infection, the presence of two of five major criteria or one major plus two minor criteria is sufficient to establish the diagnosis. A mnemonic for the major criteria is CANCER (not pleasant, but easy to remember): Chorea, Arthritis, Nodules, Carditis, Erythema marginatum, which is consistent with Rheumatic fever diagnosis.

A 59-year-old man chooses a health care provider. The health care provider discovers that the patient was recently admitted to the hospital for palpitations and was found to have newly diagnosed atrial fibrillation (AF). He was placed on digoxin (Lanoxin) and verapamil (Calan, Isoptin, Verelan) for rate control and was loaded with warfarin (Coumadin) for anticoagulation. After several trips to the anticoagulation clinic, he decided to stop taking his warfarin, as it was difficult for him to leave work during the day. He now calls the office complaining of a cold, pale, and painful right leg starting an hour ago. Which of the following is the most likely explanation for his symptoms?

Thromboembolism (not DVT) Overall, atrial fibrillation confers about a 1% annual risk for a thromboembolic event, which, among other things, may cause stroke, intestinal ischemia, renal infarcts, or a threatened limb. Specific etiologies of AF such as valvular disease are associated with higher rates of embolic events.

Which of the following diuretic agents is most likely to cause hyperkalemia? Acetazolamide, Furosemide, HCTZ, Metolazone, Triamterene

Triamterene is grouped in the category of potassium-sparing diuretics. Along with spironolactone and amiloride, these diuretic agents may cause excess renal retention of potassium. In this manner, use of these drugs may increase potassium levels and cause hyperkalemia. Acetazolamide is a carbonic anhydrase inhibitor that initially causes a moderate amount of potassium loss. In addition to being used as a diuretic, it has a role in the management of glaucoma, heart failure, and altitude sickness (it causes a metabolic acidosis that results in increased respiratory drive as the respiratory system attempts to cause respiratory alkalosis as a compensatory mechanism). Furosemide is a loop diuretic that causes a moderate to severe loss of potassium. Hydrochlorothiazide and metolazone are thiazide diuretics that cause moderate amounts of potassium loss. The majority of diuretics used in clinical practice result in hypokalemia and hyponatremia because of the loss of these electrolytes as part of their clinical utility in causing diuresis. The potassium-sparing diuretics inhibit distal convoluted tubule aldosterone-induced sodium resorption, which permits sodium loss and potassium retention. Although potassium-sparing diuretics cause weak diuresis, there is evidence showing that there is mortality benefit when these agents are used as add-on agents for chronic heart failure. These agents should be used with caution in patients who have impaired renal function with resultant hyperkalemia. These agents should also be used with caution in patients who are on medications that are known to lower glomerular filtration pressure (such as ACE inhibitors and ARBs), as these agents are known to cause hyperkalemia.

A 2-year-old boy is shot in the arm in a drive-by shooting. His brachial artery is partially transected and there is copious bleeding. Emergency medical technicians are able to control the site of bleeding by local pressure and the child stops losing blood, though he is hypotensive and tachycardic. Intravenous fluid resuscitation is urgently needed, but several attempts at starting peripheral intravenous lines are unsuccessful. Which of the following is the best alternative route in this situation?

When intravenous lines cannot be established, the preferred alternate route is intraosseous (IO) cannulation. This is done by placing a trocar in the bone marrow of a long bone. The site of choice in children is the proximal tibia; alternative sites are the distal tibia and proximal femur. Careful attention should be made in regard to IO location in order to avoid injury to the growth plate which could stunt bony growth as the child grows older.

A 45-year-old woman has a history of symptomatic ventricular couplets, for which she had been placed on amiodarone (Cordarone). A week after this intervention, she experiences a syncopal episode and is brought to the emergency department. The patient is awake and alert with a blood pressure of 110/70 mm Hg and pulse 90/min. Electrolytes are within normal limits. She is placed on a cardiac monitor and is noted to have a transient wide complex tachycardia with waves of alternating morphology. She appears to be in torsades de pointes. Which of the following is the most appropriate next step?

d/c amiodarone and observe. The patient is diagnosed with polymorphic ventricular tachycardia called torsades de pointes. This can be a fatal rhythm and is associated with a prolongation of the QT interval, which could have occurred as a result of the amiodarone administration. The appropriate step is thus to discontinue the amiodarone and observe.

A 32-year-old woman comes to the hospital for an elective repeat cesarean delivery. Four years ago she had a primary cesarean delivery for a nonreassuring fetal heart rate tracing. Two years ago she chose to have an elective repeat cesarean delivery rather than attempt a vaginal birth after cesarean (VBAC). Her prenatal course was uncomplicated except that she has mitral valve prolapse. An echocardiograph demonstrated the mitral valve prolapse but no other structural cardiac disease. Which of the following antibiotic regimen is the correct management of this patient specifically as related to her mitral valve disease?

no antibiotics are needed. mitral prolapse is moderate risk cardiac condition.

In preparation for an inguinal hernia repair, a 22-year-old man has a spinal anesthetic placed. The level of sensory block turns out to be much higher than had been planned, and shortly thereafter his blood pressure drops to 75/20 mm Hg. He looks warm and flushed, and his central venous pressure is near 0 mm Hg. He has no history of blood pressure abnormalities. Which of the following should be included in his therapy?

vasocontrictors and IV fluids. A high spinal anesthetic can produce vasomotor shock by inducing widespread vasodilation. Vasoconstrictors are the appropriate therapy, but because the capacity of the vascular tree is also increased under these circumstances, filling it up with additional volume is also helpful.

A 62-year-old man comes to the health care provider with symptoms of worsening congestive heart failure. He has a history of rheumatic heart disease as a child. Over the past 3 years he has had progressive symptoms of dyspnea on exertion, paroxysmal nocturnal dyspnea, and orthopnea. He has been maintained on digoxin, furosemide, and enalapril for symptoms of his congestive heart failure. Cardiac examination reveals a loud blowing decrescendo diastolic murmur. He has bounding peripheral pulses. Which of the following additional findings would most likely be found on physical examination? bradycardia, isolated systolic htn, pulsus paradoxus, tachycardia, wide pulse pressure

wide pulse pressure. This patient has the characteristic physical findings of an aortic insufficiency, including the loud blowing decrescendo murmur and the bounding peripheral pulses. These patients typically will have a wide pulse pressure (i.e., elevated systolic blood pressure) related to a large left ventricular volume being ejected (until left ventricular failure supervenes). The low diastolic pressure is a result of the rapid run-off from the aorta caused by the regurgitant flow across the aortic valve into the left ventricle as well as the forward flow to the aorta.


Related study sets

A.P. Industry Study Guide open to all, Please help answer the Questions.

View Set

High Rate of Natural Population Growth Case-Study: NIGER (Gabi)

View Set

TTC NUR205 MedSurgII: Chapter 25 PrepU (Hepatic & Biliary Disorders)

View Set

General Insurance Practice Questions

View Set